You are on page 1of 60





Page 2 | Text Completion | www.collegepond.com


What Is Text Completion
Text Completion questions by omitting crucial words from short passages ask the test taker to use the
remaining information in the passage as a basis for selecting words or short phrases to fill the blanks and
create a coherent, meaningful whole.

Question Structure

• Passage composed of one to five sentences


• One to three blanks
• Three answer choices per blank (five answer choices in the case of a single blank)
• The answer choices for different blanks function independently; i.e., selecting one answer choice
for one blank does not affect what answer choices you can select for another blank
• Single correct answer, consisting of one choice for each blank; no credit for partially
correct answers

Steps to solve Text Completion questions


• Read the sentence to get an overall sense of it.
• Look for Clues and Transition words.
• Predict with a word or a phrase that seems appropriate on the basis of the Clues and Transitions.

REMEMBER
For 2 and 3 blanks questions, do not assume that the first blank is the one that should be filled first;
perhaps one of the other blanks is easier to fill first. Select your choice for that blank, and then see whether
you can complete another blank.

Clues
Clues are words that help you identify who is the blank talking about, and what other information has been
provided to help you understand more about the blank.
For instance,
Many find it strange that her writing is thought to be tortuous; her recent essays, although longer than most
of her earlier essays, are extremely _________.

A. painstaking
B. tedious
C. insightful
D. sophisticated
E. clear

The blank is about her recent essays, and it’s been told to us that her writing is thought to be ‘tortuous’.
That’s our CLUE and ‘although’ is our TRANSITION WORD.

Transition Words
Transition Words are connecting words that help understand the relationship between the blank and the
information provided in the remaining part of the sentence.

Here are words that signal a contrast or contradiction:


• though / although
• but
• despite
• even though
• however
• in spite of
• instead
• nevertheless
• rather than
• yet

Page 3 | Text Completion | www.collegepond.com


• paradoxically
• ironically
• in contrast
• whereas
• while

Examples:

1. Liharev talks about being both a nihilist and an atheist during his life, yet he never does ___________ faith
in God.

A. affirm
B. lose
C. scorn
D. aver
E. supplicate

2. Ironically, the foreign affairs policies of democracies are more likely to meet with protests than similar
policies of totalitarian regimes because a democracy is _______ protest; whereas in a totalitarian regime,
no one is listening.

A. impassive to
B. indifferent to
C. imperiled by
D. sensitive to
E. inured to

Here are words that signal ideas that are similar:


• and
• for example
• furthermore
• in addition
• likewise
• moreover
• indeed
• so that
• truly
• also
• similarly

Example:

This argues well that Erikson exercised less free will than Warner; for even though Erikson was aware that
he was misdirected, he was still unable to ______ free will.

A. defer
B. facilitate
C. proscribe
D. prevent
E. exert

Here are words that signal a cause-and-effect relationship:


• as a result
• because
• consequently
• hence

Page 4 | Text Completion | www.collegepond.com


• since
• therefore
• thus
• accordingly
• due to
• for
• as

Example:

Their courage is only _______, as a small show of strength is enough to call their bluff.

A. ostentation
B. fortitude
C. temperament
D. exhibition
E. bravado

Page 5 | Text Completion | www.collegepond.com


Text Completion – Classwork Drill

1. Without any dispute, Suzy was able to ______ the state of affairs, which had been steadily spiralling out
of control.

A. reaffirm
B. belie
C. invalidate
D. fulfill
E. rectify

2. Even though the teacher repeatedly emphasized that the students be individualistic, Roger could see
through her _______ methods.

A. iconoclastic
B. disobedient
C. placatory
D. prolific
E. didactic

3. The spectators could visibly spot the ______ on Nadal’s face as he smashed his racket and marched out of
the court.

A. shrewdness
B. pedantry
C. contrition
D. benevolence
E. exasperation

4. Upon discovering fingerprints on the cup, the detective was able to show ______ proof to his client.

A. consanguineous
B. decipherable
C. homogeneous
D. discernible
E. antagonistic

5. Without _______ the fact that Snape was in love with Lily Potter, the writer said her plot would remain
incomplete.

A. mentioning
B. lamenting
C. proclaiming
D. discussing
E. rejoicing

6. Daver wasn’t very confident playing the guitar in front of his friends and family, which clearly indicated
he was a _______ with regards to playing the instrument.

A. tyro
B. prodigy
C. recluse
D. outcast
E. prodigal

7. Including ______ characteristics is something that directors often fail to do because they get caught up in
the fabrication of the story.

Page 6 | Text Completion | www.collegepond.com


A. entertaining
B. romantic
C. hypothetical
D. pedagogic
E. realistic

8. While trying to reach a settlement with the owners of the company, the union realized they had no
choice but to ______ the protest.

A. oversee
B. organize
C. stall
D. vindicate
E. ignore

9. It was apparent that Daniel had all the wrong reasons to be there; his charm was a far removal from his
true _______ intentions.

A. straightforward
B. comfortable
C. malicious
D. controversial
E. impolite

10. Even as Nigeria’s economy suffered, the president of the country stuck with his ________ outlook, stating
that oil-prices would steadily recover once the OPEC starts agreeing on production quotas.

A. clandestine
B. sanguine
C. nebulous
D. irresolute
E. vacillating

11. In the novel ‘Not without my daughter’ the protagonist Betty Mahmoody was bound to _________; the
harder she tried to escape it the tougher it got to get away from the _________.

Blank(i) Blank(ii)
A conventionalism D mediocrity
B excellence E singularity
C humility F absurdity

12. The Mumbai Indian fans were insistent that the team was not involved in the _______, in spite of all the
rumours against them. It proved to be a testing time for the ______ supporters.

Blank(i) Blank(ii)
A disarray D staunch
B scandal E opposed
C hurt F indifferent

13. South America is a continent that is always displayed in literature and film as a place of _______
adversity, however Gabriel Garcia Marquez paints ________ picture of a country of infinite potential.

Blank(i) Blank(ii)
A unremitting D a mundane

Page 7 | Text Completion | www.collegepond.com


B intriguing E an identical
C encouraging F a compelling

14. The senator, known for his poise and skill in handling delicate matters, ______ to make a snap decision
and considered it to be _______ to do it at a later date.

Blank(i) Blank(ii)
A resolved D inopportune
B grapple E apropos
C declined F warranted

15. After meticulously looking at ________ evidence expressed by the witnesses, the judge finally arrived at a
decision after tedious _______.

Blank(i) Blank(ii)
A divergent D confrontation
B arbitrary E relegation
C unanimous F deliberation

16. Mira was known to be ________ in her written works, however she was rather _______ when it came to
personal interaction.

Blank(i) Blank(ii)
A verbose D optimistic
B self-assured E disciplined
C impulsive F reticent

17. The panel thought that the speaker could have ______ many insignificant examples from the ______
speech in order to reduce the duration.

Blank(i) Blank(ii)
A added D imprecise
B revised E exhaustive
C omitted F straightforward

18. Lord Buddha _______ claims that he was a God, by stating the rumours were nothing but a ________.
Blank(i) Blank(ii)
A demonstrated D hyperbole
B repudiated E antithesis
C inhibited F paradox

19. Matt’s biggest _____ was that he underestimated his writing skills as he began to ________ his self-worth.

Blank(i) Blank(ii)
A rationale D question
B sincerity E conceal
C error F discover

20. The lyrics of the band Maroon 5 are often about the man repeatedly trying to _______ her of her
conviction of his insincerity, however the female character still remains ________ in her stance.

Blank(i) Blank(ii)
A remind D liberal
B convince E adamant
C disabuse F feigned

Page 8 | Text Completion | www.collegepond.com


21. Most patients are ________ to flock to the surgeon with the most credibility in a field; however, if
unfortunately the surgeon makes a mistake, his ________ will be affected.

Blank(i) Blank(ii)
A hesitant D persona
B loath E credibility
C predisposed F resolve

22. It is often said most writers have to navigate through a path full of thorns since they are often regarded
with ________. However EL James published the bestseller 50 Shades of Grey which was _______ enough to
_______ all doubts.

Blank(i) Blank(ii) Blank(iii)


A partiality D problematic G exculpate
B naivety E successful H assuage
C ambivalence F disparaged I belittle

23. It is a _______ that the two governing systems Marxism and Capitalism, which are ___ in several ways and
lie at divergent ends of the political spectrum; but in practice seem to be ________ authoritarian systems.

Blank(i) Blank(ii) Blank(iii)


A a proposition D unimpeachable G similar
B a paradox E antithetical H traditionalist
C an archetype F appeasable I incontrovertible

24. Just missing the Oscar creates a strange mix of _______ and _______ in the actor’s mind; although
disappointed on loosing, he is still _______ that he was nominated.

Blank(i) Blank(ii) Blank(iii)


A despondency D vagueness G euphoric
B realization E elation H tainted
C decisiveness F ambivalence I confounded

25. Although the labeling of quality products is supposed to reassure ______ consumers and help distinguish
the reliable brands from the more _____ ones, the lack of candidness of the reporting agencies has made
the rating _______.

Blank(i) Blank(ii) Blank(iii)


A timorous D precarious G monetary
B waspish E loyal H revered
C prolix F lisp I suspect

26. Rory was often labeled as harsh, sarcastic and ______. However, his wife who was often the victim of his
well-known ____ always spoke _____ things about him.

Blank(i) Blank(ii) Blank(iii)


A humiliate D diatribes G magnanimous
B bellicose E encomiums H bullish
C efficacious F tribute I prickly

Page 9 | Text Completion | www.collegepond.com


27. Rose was ________ when hurricane Sandy changed its course, since she wanted to use this as an excuse
to ______ more time. With the deadline ________ she had to deliver on her promise soon.

Blank(i) Blank(ii) Blank(iii)


A successful D refuse G deferred
B disappointed E garner H nigh
C elated F deplete I rapid

28. When he received an invite from one of the most credible agencies, the author, who had his book
rejected several times, was ______. However he couldn’t convince the editors who were skeptical about
his _______ stories in a country that boasted of its _______ ideologies.

Blank(i) Blank(ii) Blank(iii)


A elated D moral G unethical
B dumbfounded E lustful H puritan
C crying F charming I heretical

29. In the 80s most information was _______ due to scarcity of collecting empirical data. However the 90s
witnessed many technological innovations that made gathering data ________. Further, higher spend on
R&D _________ institutes to recruit data collectors.

Blank(i) Blank(ii) Blank(iii)


A unavailable D voguish G swindled
B attainable E easier H deprived
C prudent F alluring I afforded

30. Marketing plays a(n) ________ role in the success of a particular brand. Studies indicate that consumers
are likely to be more interested in a product that has a well-known celebrity endorsing it rather than a
lesser known and cheaper brand that has the same _______. The debate is how much more are buyers
willing to splurge on ________.

Blank(i) Blank(ii) Blank(iii)


A quotidian D accolades G recollection
B conjectural E constituents H delusion
C appreciable F idiosyncrasies I persona

Page 10 | Text Completion | www.collegepond.com


Text Completion – Classwork Drill Solutions

1. Solution: Answer is (E) rectify. “Without any dispute” shows that Suzy was able to Rectify the situation.
Belie (Deny) and Invalidate have negative connotations and will not fit.

2. Solution: Answer is E. “Even though” shows a contrast, which means that even though the teacher
wanted the students to think for themselves, she was being didactic.

3. Solution: Answer is (E) Exasperation (Anger). “Marching out of the court” indicates he was angry.
Contrition (Regret) and shrewdness don’t go with the blank.

4. Solution: Answer is (D) Discernible (Visible/Apparent). The fingerprints on the cup show that the
evidence was distinct. “Decipherable” is ruled out because the evidence wasn’t something that the
client had to guess.

5. Solution: The answer is (A) mentioning. The writer is simply mentioning a statement. Although
Lamenting could be an option, we realize that there is no negative connotation in the sentence.
Discussing also seems to fit in this context, but the writer is not addressing a forum here, he is simply
stating a fact.

6. Solution: The answer is (A) tyro. The clue here is ‘wasn’t very confident’ A Tyro is beginner and
contextually, a beginner may be under-confident. Both recluse (Someone who lives a simple and hard
life) and outcast (Someone who has been exiled) do not fit the answer. A prodigy would be very
confident and prodigal (wasteful) has no connection with confidence.

7. Solution: The answer is (E) Realistic. The ‘fabrication’ of the story represents an imaginative side,
which may not allow the director to create realistic features. Hypothetical is synonymous to
imaginative, where as the rest of the options do not contextually fit in the sentence.

8. Solution: The answer is (C) stall. Since it is a union, it is supposed to fight for a cause; but to “stall”
makes more sense since the words ‘no choice but to’ clearly indicate a contrast.

9. Solution: Answer is (C) Malicious. Daniel’s intentions were of a disreputable kind. The fact that “charm”
was in far removal from his actual intentions indicates the answer needs to be negative. Option (E)
impolite could also be alluring, but one cannot conclude that his intentions were rude.

10. Solution: The former half of the sentence ‘economy suffered’ denotes a negative phenomenon, which
indicates the blank needs to comprise of an answer that has the opposite connotation because of “even
as”. Clandestine means secret, nebulous means vague, and irresolute and vacillating both mean
indecisive. Option (C) Sanguine is the only option that means positive, which makes sense.

11. Solution: Answer is A, D. The clue here is— ‘the harder she tried, the tougher it got’. “Bound to”
indicates that the first blank should be filled with a negative word. ‘Excellence’ and ‘Humility are
positive so can be eliminated. As for the second blank, confirming or adhering to something may lead to
‘mediocrity’ but not to ‘singularity’ or ‘absurdity’

12. Solution: Answer is B, D. The clue here is “inspite of” indicating that the 1 st blank will be a negative
word. The fans were sure that the team was not involved. Such adamant behavior tells us they were
‘staunch’ supporters, so opposed and indifferent can be ruled out.

13. Solution: Answer is A, F. The word Adversity indicates that the portrayal was of hardship. So
‘unremitting’ (constant) fills the first blank. As for the second one, there is a contrast which is shown by
“however”. This means that it should be a word that is opposite in idea to adversity. In this case, the
word here is Compelling.

14. Solution: Answer is C, E. A skilled senator would not ‘struggle’ to make a decision. Hence he declined to
make a quick decision and thought it was Appropriate to do it at another date.

Page 11 | Text Completion | www.collegepond.com


15. Solution: Answer is A, F. In this sentence, the judge has to reach a decision, which is tedious. As for the
first blank, the word ‘meticulously’ tells us that the judge is looking not at ‘unanimous’ opinions.
Arbitrary is an extreme word to describe evidence. Hence, ‘divergent’ is correct. So he needs to take his
time. In this case, Deliberation fits.

16. Solution: Answer is A, F. We are looking at two opposite words here. This is because of the contrasting
word however. The ones that fit in perfectly are Verbose (Wordy) and Reticent (Restrained).

17. Solution: Answer is C, E. The latter half of the sentence clearly states ‘reduce the duration’. Although
added is in complete contrast to the purpose and revised could fit, but omitted is a much more precise
fit. Neither Imprecise or straightforward fit in context, whereas exhaustive which means thorough or
in-depth fits perfectly. Hence the answer is (E) exhaustive.

18. Solution: Answer is B, D. One cannot demonstrate or inhibit a claim. Repudiated means to deny. So he
Denied that he was a God by stating that it was a Hyperbole. (Exaggeration) Paradox is synonymous to
‘contradiction’ and antithesis means converse.

19. Solution: Answer is C, D. Underestimated is the key word here. Matt underestimated his skills, and so it
was an error. Conceal means to hide and discover means to find out, both of which don’t fit in context.
Hence the word ‘question’ fits perfectly, which means he doubted his self-worth.

20. Solution: Answer is C, E. The lines ‘her of her conviction’ can be confusing. One needs to simplify this
part. Disabuse (To correct) implies that he has to repeatedly correct her of her belief that he isn’t
insincere. He isn’t trying to “remind” or “convince“ her, but trying to set something straight. The second
blank is an easier one. She is Adamant (Stubborn) in her judgments.

21. Solution: Answer is C, E. In this sentence, at the first read, one may pick option A Hesitant, but one
realizes that the “however” plays a crucial role. Predisposed (Inclined) makes sense. As for the second
blank, if the expert makes a mistake his Credibility would be affected. Persona need not get affected,
and resolve does not fit in context at all.
22. Solution: Answer is C, E, H. The clue is “path full of thorns” which tells us that they are regarded with (a
negative option) ambivalence (mixed feelings / doubts). “Bestseller” indicate that for the Second and
Third blanks, Successful and Assuage (mitigate/reduce) fit in well.

23. Solution: Answer is B, E, G. The key phrase ‘lie at divergent ends’ shows that blank 2 will require the
word ‘antithetical’. ‘But in practice’ introduces the contrast. Hence blank 3 will have ‘similar’. Blank 1
therefore will have ‘a paradox’. Such contrast can only be a paradox. It cannot be ‘hypothesis’ which
means a tentative explanation given as a starting point for further research. An ‘archetype’ means
typical. It cannot be typical that such contrasting systems would turn out to be similar.

24. Solution: Answer is A, E, G. The key phrase ‘a strange mix’ shows that the first and second blanks have
to be opposite to each other. In this case, Despondency (Sadness) and Elation (Happiness) fit perfectly.
The word ‘although’ indicating contrast shows the word in blank 3 would be a contrast of
‘disappointed’ Therefore, Confounded (Confused) and Tainted (Spoilt) do not fit in. So Euphoric
(Overjoyed) makes sense.

25. Solution: Answer is A, D, I. The consumers who need to be reassured are the timorous (lacking
confidence) ones so that they can distinguish between the reliable and Precarious (Risky) brands. As
for the third blank, the sentence mentions that there is lack of frankness so the rating is Suspect
(Dubious).

26. Solution: Answer is B, D, G. The first blank has a negative connotation which will be Bellicose
(Aggressive). In the second blank, the word Victim assures that it is another negative term. In this case,
Diatribes (Verbal attacks) fills the blank. As for the third blank, the ‘However’ indicates that the wife
spoke Magnanimous (Noble) things about him.

27. Solution: Answer is B, E, H. Rose wanted to use the hurricane as an excuse. Therefore, the fact that the

Page 12 | Text Completion | www.collegepond.com


hurricane has changed its course could only lead to ‘disappointment’ and not ‘elation’. The hurricane
could have helped her garner (buy) time (blank 2) as the deadline was nigh (Close). Deferred means
put off to a later time, which does not fit in context.

28. Solution: Answer is B, E, H. The phrase ‘rejected several times‘ indicates that he would be surprised or
‘dumbfounded’ (blank1) when he received an invite. The “However” in the second line brings about a
shift. The phrase ‘editors were skeptical’ shows that something about the stories was not right for
them. Lustful (Immoral) fits well. As for blank 3, it will be opposite in nature to blank 2. If he couldn’t
sell the book due to its ‘lustful’ nature, then the ideologies could be described as ‘Puritan’ or based on
purity of thought and conduct, since that is the only one that represents a contrast.

29. Solution: Answer is A, E, I. Due to scarcity of data, information was Unavailable. Later because of
advancements, it made collection Easier. The last blank is Afforded (Allowed).

30. Solution: Answer is C, E, I. The first blank is Appreciable. Quotidian is synonymous to day-to-day and
Conjectural means Speculative, both of which do not fit in with the meaning of the first blank. The
second blank Is Constituents, and not accolades, because it isn’t about awards or personal
idiosyncrasies (a way of thought peculiar to an individual), but about the constituents or the
characteristics of the product. The last blank is Persona, because the question asked is how much are
shoppers willing to spend on a Celebrity endorsement. Both Recollection and Delusion do not fit in
context.

Page 13 | Text Completion | www.collegepond.com


Secure your
Dream University
with Collegepond’s Counselling Services

360° Career Profile


Counselling
Counselling Building Feedback
Schools and SOP, LOR
Program Selection and Resume 4.1 Counsellor Availability

4.2 Career Counselling


Converge Application
Online Portal Assistance 4.3 Financial Counselling

4.3 Seminars
VISA
Scholarships Counselling 4.3 University Selection

4.4 Resume Preparation


Interview Financial
Preparation Planning 4.4 Application Counselling

4.4 Recommendation Prep


Pre-departure Post-landing
4.6 Statement of Purpose
Orientation Services

022 - 2610 3135 | 022 - 2619 5832 Collegepond


info@collegepond.com www.collegepond.com
Text Completion Homework Drills

1. During the winters, Lake Tahoe freezes so ______ that one can casually stroll off the land mass without
knowing that you are standing over a water body.

A. everlastingly
B. indistinguishably
C. erratically
D. unpredictably
E. obscurely

2. Trying to remember the multiple rules of the exceptions to the French language can be so _________ that
even those with a keen grasp of the language need to have a manual handy to refer to every time they
are stuck.

A. monotonous
B. contentious
C. artless
D. frustrating
E. somber

3. The singer-cum-songwriter never courted fame. His rugged modernism seems to disregard rather than
to ______ his audience.

A. defy
B. rebate
C. enlighten
D. woo
E. intimidate

4. The class of 2010 presented their final thesis on the study that the general masses were __________ the
propagandistic influence of the media, but their supervisor contradicted them by stating that mass
communications rarely produced pronounced changes in social behavior.

A. unimpressed by
B. scornful of
C. susceptible to
D. unaware of
E. contend with

5. Though the recently elected mayor was not _________ the arguments in favor of the new bridge
construction, he vetoed the proposal.

A. optimistic about
B. unsympathetic to
C. elated about
D. sanguine about
E. convinced by

6. For majority of the first half of the twentieth century, the study of arts at the university was in ________
state, in spite of the presence of several dignitaries.

A. a scintillating
B. an abysmal
C. a contentious
D. a combustible
E. a veracious

Page 14 | Text Completion | www.collegepond.com


7. It was a pity that the artist once considered to be the greatest living expert by his contemporaries was
now buried in _________ with no accolades to his name.

A. specify
B. negligible
C. extol
D. sufficiency
E. obscurity

8. With the discovery that the level of carbon emissions are crossing the threshold safety levels, the global
protest by environmentalists world over was _________, even more so after they repeatedly mentioned
the adverse effects of further polluting.

A. injudicious
B. premature
C. fortuitous
D. imminent
E. improbable

9. In his initial works, Mitch Albom made death a/an(i) ________ theme. His early critics inferred that he
had a predilection for focusing on (ii)_________ themes.

Blank(i) Blank(ii)
A pivotal D Recondite
B non-existent E Uncomplicated
C obscure F Morbid

10. Multinational companies these days are largely ________ to fresh employees because their existing work
culture fails to make relevant connections to the cultural references new employees understand. It is
no wonder that many employees regard job training as a/an __________ ritual.

Blank(i) Blank(ii)
A equipped for D covert
B opaque to E sterling
C essential for F painstaking

11. Paradoxically, the painter so concerned about__________ was _________ with paint and varnish. His
material, often being of higher quality, cost him a fortune at the time.

Blank(i) Blank(ii)
A integrity D acquisitive
B extravagance E puritanical
C disapprobation F profligate

12. The recent survey in the National Geographic is surprisingly ____________. It makes most other longer
surveys on the effects of lost habitat seem _________.

Blank(i) Blank(ii)
A twisted D redundant
B impartial E cogent
C Money-grubbing F untimely

13. Professions like engineering, medicine and architecture are well known for their regimented work
schedules. It is a myth that creative folk do not have to abide by the same discipline that governs the
rest of the professions; that they reject (i)___________. This is of course not true as most artists work just
like the rest of us (ii) __________ day by day based on the demands of their work.

Page 15 | Text Completion | www.collegepond.com


Blank(i) Blank(ii)
A latitude D foolhardy
B routine E ploddingly
C Critical F sporadically

14. Knowing just how _________ the surgery is, it is astonishing that a lot of the doctors have failed to grasp
the challenges involved. In spite of that, doctors persist in adding new procedures that are perceived as
_________ to those who actually comprehend the enormity of the procedure.

Blank(i) Blank(ii)
A irrelevant D quixotic
B dire E meticulous
C Critical F twisted

15. The belief that the Police Commissioner might turn _________ after taking office led to the appointment of
ethics officers at several layers of the government.

A. scrupulous
B. ensconce
C. venal
D. perplex
E. Imitation

16. Mathematicians have a distinct sense of beauty, they strive to present their results and ideas in a clear
and compelling fashion, dictated by ______________ as well as by logic.

A. urge
B. aesthetics
C. commitment
D. methodologies
E. intellect

17. The ________ of Sia Pepper as one of South America’s literary giants has thrown the spotlight on her
peripheral activities and has thus served to ______ her efforts as an engineer.

Blank(i) Blank(ii)
A stigmatization D foreground
B lionization E overshadow
C marginalization F undistinguished

18. Conceited with a tendency towards recklessness, Rahul could not handle fame, the more his ________ as
an artist increased, the more _________ his life became.

Blank(i) Blank(ii)
A Abstinence D Tumultuous
B Notoriety E Opportune
C Eminence F Dispassionate

19. The painter’s style of working is ________. He creates drama on the canvas; his style _________ the artistic
methodologies normally used to create art.

Page 16 | Text Completion | www.collegepond.com


Blank(i) Blank(ii)
A monotonous D jettison
B startling E experiment with
C honoured F refine

20. Inept managers refuse to admit their faults and implicitly claim _________, thus embarrassing the
company and undermining the policies, which has several mechanisms of rectification based on the
fact that anyone could go wrong and think it is _______, and seek the benefit of the _________ process

Blank(i) Blank(iii) Blank(iii)


A infallibility D discomfiting G an adaptable
B exemption E beneficial H a remedial
C detachment F forthcoming I an unwise

21. The general notion that pop music is inferior to classical music is now obsolete. Yet the change of
attitudes may be somewhat ________. Though it is now academically respectable to analyze pop music,
the fact that intellectuals feel pressured to rationalize their own _______ Taylor Swift or Katy Perry songs
reveals, perhaps unwittingly, their continued _________ the old hierarchy of high and low culture.

Blank(i) Blank(ii) Blank(iii)


A counterproductive D penchant for G aversion to
B underappreciated E distaste for H investment in
C exaggerated F uninterested in I misconception of

22. Elon Musk’s Tesla is at little risk of colliding in the Earth’s orbit; however given the statistics of new
devices to sail each year, the skies in the future are likely to be less __________.

A. crowded
B. impervious
C. fortified
D. polluted
E. benign

23. While the new intern was undoubtedly __________ to the art director, such unquestioning compliance
and sycophancy did not help him rise beyond the role of an intern.

A. glib
B. insolent
C. benevolent
D. obsequious
E. deferential

24. While the New Yorker is one of the critically acclaimed magazines, so much time is spent describing a
characters’ clothing that the reader often feels that such __________ concerns have overridden any more
direct literary aims.

A. didactic
B. syntactical
C. tangential
D. sartorial
E. inane

25. Evolution ensures that every successful species can _________ its innate capability for reproduction and
growth with the limitation that arises through its co-relation with nature.

A. amplify

Page 17 | Text Completion | www.collegepond.com


B. substitute
C. assemble
D. surpass
E. reconcile

26. Though India’s economic progress has traditionally been viewed as the ________ for penury in rural
areas, the environmental side effects are becoming worrisome.

A. culprit
B. receipt
C. panacea
D. explanation
E. asylum

27. In the uncensored version of her memoir, Elizabeth Stone remarked openly on the corruption in her
homeland; her analysis being so ________ that her editors were afraid that they would hurt the country’s
reputation if not held back.

A. effervescent
B. acerbic
C. cautionary
D. laudatory
E. clement

28. With the rate of technological ___________ gaining momentum, we now think of a laptop, ________ after a
few years, the dilemma of how to discard worn out appliances will be no laughing matter.

Blank(i) Blank(ii)
A affordability D unfathomable
B complication E well known
C obsolescence F outdated

29. While fans attributed the singer’s success to his manager’s ________, his close friends and family came to
realize that his success was more dependent on the manager’s contacts than to any ___________ that the
manager was supposed to have.

Blank(i) Blank(ii)
A perseverance D obduracy
B popularity E crotchet
C prescience F foresight

30. Nadal has notable tennis skills, but as an orator he is far less _________ than his opponent, whose
elocution skills are ________.

Blank(i) Blank(ii)
A adroit D unpretentious
B implausible E specious
C discursive F breathtaking

31. The iphones and ipads that mushroomed during the last decade were _________, far too ________ for the
lower income class populace.

Blank(i) Blank(ii)
A opulent D pastoral
B multifaceted E exorbitant

Page 18 | Text Completion | www.collegepond.com


C baffling F mundane

32. The French school of design was _________ several twentieth century artists, so much so that today the
term ‘academic art’ has become a __________.

Blank(i) Blank(ii)
A influential among D pejorative
B ridiculed by E quandary
C visited by F misnomer

33. As a respected film critic, Rajeev Masand has penned a ____ book about filmmaking culture in India in a
disorderly mannerism. His light humour is _______ the style of movie criticism. But Masand can’t claim to
be _________, he has confined himself to information available in his personal collection.

Blank(i) Blank(ii) Blank(iii)


A deft D wonderfully at odds with G congenial
B pretentious E heavily influenced by H accessible
C all-inclusive F largely supplanted by I thorough

Page 19 | Text Completion | www.collegepond.com


Text Completion Homework Drills Solutions

1. Solution: Answer is B. The only blank that fits here is option B indistinguishably, which means it is difficult
to tell apart. If you look at the sentence, it mentions that you would not know that you are walking over a
water body, which indicates that the water freezes in such a subtle manner one would not notice or
‘perceive’ the transition.

2. Solution: Answer is D. ‘Even those’ indicates that understanding French is difficult for someone who has a
good knowledge of the language. The word that fills in the blank in this case is Frustrating. Monotonous
means boring and contentious means debatable, artless means open or honest and somber indicates
earnest. None of these fit in context.

3. Solution: Answer is D, woo. We are looking for a word that is the opposite of disregard or defy. The word
that does that is Woo (Persuade). Rebate is a refund, enlighten means to illuminate, both of which could
make sense if it wasn’t for the former half of the sentence.

4. Solution: Answer is C. The word contrast word ‘but’ divides the sentence into two parts. The second part
indicates that media hardly plays a role in social attitudes. Therefore the first part has to contrast with this
and state the public is affected by media. Hence, the option that fills the blank is C. Susceptible to
(Vulnerable).

5. Solution: Answer is B. The contrast word ‘though’ divides the statement into two parts. The second part
states that he Vetoed the proposal (voted against it), So the previous part should contrast this and state
that he was not against it. Option B, unsympathetic to, shows that.

6. Solution: Answer is B, an abysmal. The key words being ‘in spite of’, which indicates that the answer should
be the opposite of what dignitaries connotes. Hence the university is supposed to have a negative adjective,
which is why abysmal (pathetic) is a perfect fit. Scintillating means sparkling, which doesn’t fit in context
because of the latter half of the sentence, contentious means controversial, which could fit but isn’t
corresponding to the second half of the sentence. Combustible means inflammable and veracious means
authentic, both of which do not fit the context at all.

7. Solution: Answer is E. Keywords- ‘it was a pity’ that indicates that the answer should be in contrast to
‘greatest living expert’. The word that does so is ‘obscurity’. Extol means to praise enthusiastically, and
specify, negligible and sufficiency do not fit in context.

8. Solution: Answer is D. The word ‘even more so’ emphasizes the part of the sentence that comes before it.
Therefore, the correct answer choice has to justify the ‘uproar’. Hence, Option D imminent is correct.
Injudicious means means imprudent or unwise, which does not fit in context. Fortuitous (unpredictable)
and improbable both don’t match with what the blank is supposed to mean.

9. Ans: A and F
The word “predilection” means a liking for something in particular. We have to find a pair that makes
sense, and non-existent and obscure do not match with any of the options for the second blank. Pivotal
means centric, which can be paired with morbid (death-obsessed) to highlight the writer’s theme.

10. Ans: B and F


Keywords are ‘fails to make connections’. Fail denotes a negative connotation; hence we have to look for an
option that corresponds. They are Opaque to (Impervious) fits appropriately. This is why many employees
view training to be Painstaking. Sterling means splendid and covert means secret.

11. Ans: B and F


The keyword ‘paradoxically’ shows that we need two contrasting ideas. If the “painter” who was cautious
could also be wasteful (Profligate) with “paint” and “varnish”, then it would be paradoxical. Integrity is out
of context and disapprobation means an expression of strong disapproval. Acquisitive means excessively
interested in acquiring material things, and puritanical means prudish, neither of which fit in context.

12. Ans: C and D

Page 20 | Text Completion | www.collegepond.com


Both sentences reflect the contrasting qualities of the same survey. Hence the correct answers would have
to be a pair of antonyms and opposite in connotation. The keywords ‘most other longer surveys’ shows
that the survey mentioned in the first part is also ‘long’. Therefore, C Comprehensive is correct. It is ‘long’
used positively. For the second blank, the “other longer surveys on lost habitats” would be Redundant
(negative in connotation). “Cogent” means convincing which does not contrast with comprehensive.
Untimely which means not at the right time does not fit in this context.

13. Ans: B and E


This passage conveys the sense that artists are like everyone (hint: ‘of course not true’). ‘The myth’
indicating a contrast in the latter half of the sentence. The correct answer for the first blank is B.”routine
(habit)”. Latitude means scope for freedom or action or thought, which does not fit in context. Money-
grubbing could fit with the first blank, but makes no sense in connection to the second one, which is clearly
a spectrum of quantitative work, not the material benefits. The correct answer for the second option is
Ploddingly. Foolhardy means recklessly bold or rash, and could fit in connection to an artist but does not fit
the context above. Sporadically means occasionally or at irregular intervals, which is the opposite of what
we are looking for.

14. Ans: C and D.


Keywords—It is astonishing that a lot of the doctors have failed to grasp the challenges’----shows the
project is filled with serious issues, hence “Critical” fits in the first blank. Despite it being difficult and not
handled well, the doctors continue to add new complexities, so it could be said that they were idealistic.
This is why Quixotic would fit the second blank.

15. The answer is C. venal, which means corrupt. Scrupulous means extremely honest which is the opposite of
what we want, since ethics officers have been appointed for a purpose. Ensconce means to settle someone
in a comfortable place, which is out of context. Perplexed means surprised and imitation means something
fake, both of which do not fit in context.

16. Ans: B
The word beauty in the first half of the sentence clearly indicates the results and ideas are supposed to
comprise of that element. Neither Option A. urge nor C. commitment, convey an idea of beauty. Option D.
methodologies and Option E. Intellect could fit had it not been for the first half of the sentence. Hence the
only option left is Aesthetics, which is the right answer since it means a set of principles concerned with the
nature and appreciation of beauty.

17. Ans: B and E


Sia Pepper’s success as a writer has led to her being under spotlight, hence “lionization” (or adoration) fits
in the first blank. Neither the words stigmatization nor marginalization fit in this context, since both
represent a turning away from her, as opposed to calling attention to her. ‘Peripheral activities’ means
outlining, or of
Secondary importance which suggests that writing was not Sia Pepper’s primary profession. Being lionized
as a writer has overshadowed her primary profession as an engineer.

18. The correct answer for the blanks is C. Eminence and D. Tumultuous. The first few words ‘conceited’ and
‘recklessness’ are both negative, which tells us that the second blank would be negative too. The first blank
is in direct relation with the second, which works as a hint that it should be something in proportion to the
second and related to fame. While notoriety (the state of being famous or well known for some bad quality
or deed) could increase, it is not as clear a sign of success as eminence is. Abstinence is the practice of
retaining from indulging in something, which is the opposite of what the blank indicates. Opportune means
especially convenient for a particular action or event, and dispassionate means someone who is not
influenced by a strong emotion, and so able to be rational and impartial, both of which fit in context.
Tumultuous means confused and disorderly, which is a perfect fit for the second blank.

19. Solution: Answer is B and D. The keywords are—‘he creates drama on canvas’. Hence for the first blank, the
word choice that fits in the context is ‘startling’. The second blank extends the effect of the keywords
‘creates drama’ and the key words are ‘normally used’. So in order to ‘create drama’, the artist needs to do
away with ‘normally used’ techniques. So ‘Jettison’ (overthrow) is the correct choice.

Page 21 | Text Completion | www.collegepond.com


20. Solution: Answer is A, D and H. For the first blank, the keywords are ‘refuse to admit their faults’, this
means that they think they are Infallible (Cannot be blamed). Understanding context: keywords
“embarrassing the company” have negative connotation, so ‘self-correction’ would be difficult. Hence,
Discomfiting (Embarrassing) is the correct choice. The sentence talks about corrective action, hence the
third blank should be remedial.

21. Solution: Answer is C, D and H. For the first blank, the discourse marker ‘yet’ strikes a contrast with the
term ‘obsolete’. Hence to question the obsolete status, the option ‘exaggerated’ is the best choice.
‘Counterproductive’ and ‘underappreciated’ are out of context as there is no reference to productivity or
appreciation. If the idea that popular culture is inferior to high art is not obsolete, then people would feel
guilty about liking popular art. The keyword ‘rationalize’ is apt in this case. Hence the choice for blank 2 is
D. ‘Penchant for’. Since the idea still persists, it ‘reveals’ that they are loyal to the idea of preference to high
culture over low culture. ‘Aversion to’ and ‘misconception of’ give the opposite effect.

22. Solution: Answer is E. The discourse marker ‘however’ introduces a contrast to the original idea. The
original idea is ‘little risk’, ‘but’ the increased number of devices may increase the traffic, so the effect would
be ‘harmful’. The word less again changes the direction, so the correct answer would be an antonym of the
word ‘harmful’, So benign (harmless) is the correct choice. ‘Crowded’ is the opposite of what is needed,
impervious (impenetrable) and fortified (protected) are both out of context. Polluted could fit if it wasn’t
for ‘colliding’ mentioned in the former half of the sentence, which makes it out of context.

23. Solution: Answer is D. Keywords are ‘unquestioning compliance’ and ‘unthinking sycophancy (praising
people in authority in a way that is not genuine)’, so the answer choice would be a synonym of submissive
and servile (showing excessive willingness to please others). Hence, Obsequious (obedient or attentive to
an excessive or servile degree) is the correct choice. Glib means fluent but insincere and shallow, which is
the opposite of what is required in the first blank. Insolent is rude and benevolent is well meaning and
kindly, while deferential is respectful. Deferential could fit, however it is not as much of a perfect fit as
obsequious is.

24. Solution: Answer is D. Contextually, sartorial is a term that is related to tailoring, clothes or style of dress.
The keywords are “avid descriptions of his characters’ clothing”. Hence, sartorial fits into the context over
other choices. Didactic means a story with a moral, syntactical is something to do with syntax, tangential is
synonymous to irrelevant and inane is something frivolous. Although inane could fit it does not come as
close to the meaning as sartorial does.

25. Solution: Answer is E. Keywords—‘Successful species’. For this sentence, at the first go, amplify looks like
a good option. If the species is ‘successful’, the capacity for population growth must be ‘amplified’. But, the
sentence further correlates ‘innate capability’ with ’limitation’. ‘Substituting’, or ‘assembling’ or innate
capability with constraints won’t make the species successful. ‘Surpassing’ leads to grammatically incorrect
construction. Hence, ‘reconcile’ is the correct option. It is also semantically correct as only when you
‘reconcile’ (make one consistent with another) these two different elements, can the species be considered
‘successful’.

26. Solution: Answer is C. The word ‘Though’ marks a contrast with the part of the sentence after the comma.
Keywords are “side effects are becoming worrisome” Which has a negative connotation. Hence the word in
the blank should be a positive word. Hence Panacea (medicine/cure) is the correct choice.

27. Solution: Answer is B. Keywords — “remarked on the corruption” and “they (comments) would hurt”, so
the answer will be a negative word. ‘effervescent’ (enthusiastic), ‘laudatory’ (flattering) and ‘clement’
(mild) are positive and can be eliminated. ‘Cautionary’ (related to forewarning) is out of context. Hence,
‘acerbic’ (scathing) is correct.

28. Solution: Answer is C and F. Keywords being--- “discard worn out appliances” shows that a PC is no longer
considered necessary; and the answers have to reflect that. The part after the comma divides the sentence
into two parts, the second part simply extends the meaning of the first one. So the answer choice must be
synonymous. This makes ‘obsolescence’ (the process of becoming outdated’ and ‘outdated’ correct.

Page 22 | Text Completion | www.collegepond.com


29. Solution: Answer is C and F. The word ‘While’ introduces a contrast between the perception of his fans and
his close relatives, so ordinarily the answers would be antonyms. However, the words ‘this success owed
more to the manager’s contacts than to any’ change the direction, hence, the answers would be synonyms.
In this case, C ‘Prescience’ (foresight) and F ‘Foresight’ are correct. Perseverance could fit in the first blank
but has no synonym for the second, popularity could fit in the first line but does not make sense
considering the latter half of the sentence, and obduracy (stubbornly refusing to change one’s opinion) and
crotchet (an unfounded belief or notion) could fit in the second blank but have no synonyms in the first,
hence taking into account the first half of the sentence, lack in context.

30. Solution: Answer is A and F. The keywords “but as an orator she is far less “ marks a contrast with the
previous part of the sentence—“political ambassador has considerable political abilities”. There is a
comparison between two types of skills- Political and oratorical. So, the choice has to be a word related to
skills, in this case, option A. Adroit(skillful). Further, the contrast is extended as there is comparison of his
oratorical skills with that of his opponent. So the answer choice has to be an antonym equivalent to the
negative connotation in blank one. In this case, it would contrast with ‘less adroit’, hence the positive word
‘breathtaking’ is contextually correct in terms of skills.

31. Solution: Answer is A and E. The sentence construction ‘ far too _____for the standard middle class shows
comparison—of people with average means to those with not so average means (unstated comparison).
So, option A Opulent (Lavish)is correct. Such Lavishness would be costly, hence option E expensive is the
correct choice for blank 2. Multifaceted could fit in the first blank if it wasn’t for the latter half of the
sentence, while baffling is completely out of context. Pastoral means clerical and mundane means
monotonous, both of which don’t fit in context.

32. Solution: Answer is B and D. The construction ‘So much so that’ divides the sentence in to two parts that
have a cause-effect correlation. So the answer choices will have to support this co-relation. The only pair
that does this is Option B ‘ridiculed by’ and ‘pejorative’. Because of ‘ridicule’ the term ‘academic art’
becomes a pejorative (insult). Hence these choices are correct. Quandry means a dilemma and misnomer
means an inaccurate name or designation, both of which are out of context.

33. Solution: Answer is A, D and I. First blank--The keyword ‘respected’ has a positive connotation. So, the
correct choice will be a positive word. ‘Pretentious’ can be eliminated. The keywords ‘confined himself to
information available in his personal collection’ shows that the book was not ‘comprehensive’. Hence the
answer is ‘deft’. For second blank—‘light humour’ is contextually a contrast with the keyword ‘criticism’.
Hence one cannot be influenced or supplanted by another. So, option D, ‘wonderfully at odds with’ is the
correct answer. Third blank---keyword ‘confined’ indicates cannot be ‘thorough’.

Page 23 | Text Completion | www.collegepond.com


Where our Alumni
are Working

022 - 2610 3135 | 022 - 2619 5832 Collegepond


info@collegepond.com www.collegepond.com
Text Completion 1 blank drill

1. Cricket is the most followed sport in India in almost all respects, whereas in Australia fans are evenly
_________ among a few different sports.

A. considered
B. unconcerned
C. appreciated
D. measured
E. apportioned

2. Several Greek philosophers argued about the shape and the positioning of Earth to be a form of mass
__________, and that only a detailed explanation with proofs could allow the ignorant people to
understand that Earth is round and revolves round the sun.

A. innovation
B. delusion
C. tragedy
D. metamorphosis
E. frenzy

3. In the 1970s, the idea that everything would be controlled by a small electronic device of the size of
palm would have sounded _________.

A. predictive
B. incongruous
C. presumptuous
D. preposterous
E. overwhelming

4. The World Wars caused a great destruction to life and property, but the nuclear bombs dropped by
USA on Japan had more ___________ impact, for almost 50 years the people in that region were affected.

A. distinguished
B. enduring
C. ephemeral
D. insidious
E. confounding

5. Many ___________ people believe that changing their name by adding or removing an extra alphabet in
their name, would bring them more success and fame as it did for some famous personalities like
Hrithik Roshan and Ajay Devgn.

A. erudite
B. maverick
C. superstitious
D. muddled
E. orthodox

6. The US university ask for loan papers before admitting the student; whereas the Indian banks, in turn,
ask for the admit letter first to issue a loan document; such is the __________ of application process.

A. fickleness
B. irony
C. circle
D. fortuity
E. satire

Page 24 | Text Completion | www.collegepond.com


7. Apart from their conventional knowledge, it is important for the engineers to remain _________ the latest
developments in their field and update their knowledge regularly.

A. akimbo to
B. abreast of
C. obtuse to
D. deferential to
E. contemptuously to

8. After the formation of Pakistan and war in the middle east, many families, especially Parsi families,
_________ the Indian peninsula, with a hope of living peacefully.

A. annihilated
B. inundated
C. ostracized
D. shunned
E. rebuked

9. The recent bomb-blasts in London were a _________ to a terrible year for the civilians, recently wracked
by civil strife and Brexit.

A. purgation
B. coda
C. prologue
D. homily
E. frenzy

10. Due to rampant slavery and poverty in Zimbabwe, the residents are _________ suffering, casting a blind
eye to the hopelessness around them.

A. inured to
B. effusive towards
C. maudlin over
D. spared from
E. authoritative over

11. Stark’s victory in the Battle of the Bastards followed a dispute and struggle for the position of power
among the other Houses in the North, but eventually the Northerners elected Jon Snow as their leader
and _________ into a surprisingly compatible team.

A. ruptured
B. agglomerated
C. amortized
D. syncopated
E. coalesced

12. The author decided to purposely introduce __________ in the story; for example Mahatma Gandhi
leveraged social media platforms like Facebook to call for a boycott of foreign goods.

A. irony
B. satire
C. platitudes
D. neologisms
E. anachronisms

13. There lies an ancient rock on the southern shores of India which doesn’t even move an inch from its
position though situated on an inclined plane. It presents a true _________: none have been able to

Page 25 | Text Completion | www.collegepond.com


decode the reason behind its rigidness and as to why doesn’t it move under gravity or through the
forced means.

A. jewel
B. stanza
C. quandary
D. enigma
E. incitement

14. The suffering experienced through the breakups find a way into the writer’s stories; and so does the
experience of losing his parents at a very young age ___________ his story writing.

A. inform
B. postpone
C. portend
D. imitate
E. gainsay

15. The candidate who had listed German as his language skill in his resume, accidentally went to the
wrong room for his interview, thereby creating _________ situation where he began talking in German in
front of Spanish panellists.

A. a foul
B. an incorrigible
C. a risible
D. a perfunctory
E. an arch

16. The engineer told his boss to design a new innovative product, but his suggestion to manufacture was
____________- apparently, he naively thought that just putting forward the idea will lead into the product
development without Research and Development.

A. intricated
B. cordial
C. soothing
D. facile
E. unblemished

17. Instead of a quiet Sam, the girl decided to spend the rest of her life with Kevin, who is more _________
and thus, an extrovert.

A. dogmatic
B. convivial
C. sanguinary
D. insipid
E. trendy

18. The world is on the transformation from gasoline cars to electric cars, which clearly demonstrates that,
when the __________ proves more successful or at least comparable with the present time-honored, the
government will eagerly accept the new changes.

A. upheaval
B. thrilling
C. novel
D. conventional
E. disputable

Page 26 | Text Completion | www.collegepond.com


19. Being broken deep inside after ordering the killing of her own son Armendra Baahubali, the bereaved
Sivagami, finally burst into __________; she couldn’t maintain her composure.

A. hysterics
B. gaiety
C. flames
D. merriment
E. eulogy

20. The World Wrestling Federation famously known as WWF, has for decades now, employed a bifurcated
aesthetic: like boxing, they try to represent real fighting which gives authenticity to their show, but
revenues dictate that they simultaneously employ subtle _________, like creative storylines, scripted
matches, etc that makes the show much more attractive than plain boxing.

A. artifices
B. sentiments
C. creation
D. artistics
E. gimmicks

21. The preachers of religion with their dogmatic nature have actually alienated educated citizens because
they fail to realize that their _______ does not inspire common people to blindly follow their religion.

A. shrewdness
B. pedantry
C. frugality
D. pulchritude
E. acumen

22. The manager focused so intently on the petty, logistical details of the project that the attention to
_________, had supplanted more worthy targets.

A. irrelevancies
B. tactics
C. minutiae
D. strategy
E. peccadilloes

23. It is said that humans have developed from monkeys and as an evolutionary adaption, humans possess
the smartest brains of all the animals and this is a safeguard that ____________ their success as a species.

A. replicates
B. alleviates
C. swamps
D. guarantees
E. enhances

24. The Corleone family made a ______ choice to double-cross the most powerful gangster turned politician
and they paid the price for that dangerous decision.

A. saga
B. cious
C. parlous
D. suicidal
E. semiotic
F. propitious

Page 27 | Text Completion | www.collegepond.com


25. In the movie, a valiant effort was made by the heroes to stop the train by making it collide with a huge
rock, so that train loses its momentum and slowly _________ to a stop.

A. careened
B. chastised
C. trundled
D. tarried
E. exacerbated

26. The Bhagavad-Gita, the holy book of Hindus, is of ___________ nature, advising people to be honest and to
believe in their work without expecting results.

A. a magnanimous
B. a sceptical
C. an irate
D. a didactic
E. a paradoxical

27. It was an uphill task to find the referee for the football match as the nature of the game demanded that
on some controversial decisions the referees need to be ________

A. lucid
B. droll
C. bigoted
D. disinterested
E. solipsistic

28. When John was finally acquitted after 25 long years, instead of grieving about the pain and agony he
suffered in these years, he adopted a mood of exaggerated hilarity, almost as if the decision came as a
__________ surprise.

A. unctuous
B. risible
C. grasping
D. imprudent
E. plaintive

29. The overnight success of the simple businessman _________ the years of toil required to achieve it, from
months of wrong investments in unsuccessful ventures to working hard to prevent bankruptcy of the
firm today known for its world-class products.

A. amends
B. reaffirms
C. belies
D. furnishes
E. stimulates

30. Buddhists living in Pakistan, who accepted Islam, were not allowed to fight from the Pakistan side in
the post-independence war, as the government to label them “__________ anti-nationals” and claimed that
those who can’t stay true to their own religion cannot fight whole-heartedly for our victory.

A. premature
B. gory
C. dogmatic
D. atheist
E. communist

Page 28 | Text Completion | www.collegepond.com


31. Archaeologists study human history and prehistory through the excavation of sites and the analysis of
artefacts and other physical remains, but most of their research occurs in libraries, and most of their
discoveries are ___________ from tomes which are sometimes as old as the artefacts.

A. perused
B. embellished
C. gleaned
D. convoluted
E. explicated

32. Sachin Tendulkar is known to gently ________ his junior cricketers who don’t work hard, but because of
his generally amiable personality, he refrains from punishing anyone.

A. despise
B. ameliorate
C. pillory
D. penalize
E. chide

33. One cannot possibly imagine what would happen if social apps like Facebook, Snapchat, Instagram are
banned; for a change the teenagers may resort to doing productive things in their free time due to the
scarcity of mobile apps where they usually can _____.

A. thrive
B. contemplate
C. loaf
D. prowl
E. flirt

34. With the nuclear powers in the world rising, the ___________ of nuclear holocaust has become a hidden
fear in the citizens all over the world and especially non-nuclear countries.

A. spectre
B. annihilation
C. savagery
D. explosion
E. existence

35. The Indian justice courts ___________ those who see paradigms for “triple talaq”, describing that it doesn’t
make sense to divorce a woman by saying her talak three times orally and it should be abolished given
that it is also banned in Muslim majority countries.

A. conspire with
B. block out
C. rail against
D. skitter around
E. pass over

36. Always keep yourself away from the __________ of people who pretend to be your well-wishers, because
the actual ones would abruptly give you an honest advice even if you would feel terrible about it.

A. torpor
B. regalia
C. babble
D. puffery
E. confabulation

Page 29 | Text Completion | www.collegepond.com


37. Fed up by the notion of Indian parents of making their kids Engineers or doctors, Sam cursed the
educational system and __________ the lack of support for experimental art in this country.

A. demean
B. decry
C. tarnish
D. delineate
E. delay

38. Middle class people in India have developed a notion that doctor prescribes them costlier medicines
and treatment just to earn the money, and since these people cannot afford it, they stop the treatment
midway when they feel slightly better. However, this practice of incomplete course of treatment spares
the most ______________ bacteria and viruses which may cause the disease again.

A. ubiquitous
B. robust
C. benignant
D. scandalous
E. jejune

39. Even though son of the rich businessman Paresh Bambani was expected to be a spoiled brat, he came
out as a ______________: a preternaturally intelligent human being who lived up to the expectations.

A. prodigy
B. cynic
C. dynamos
D. Adolescent
E. crackpot

40. When the famous Ed Warren was trying exorcism in a room filled with 3 people, the other 2 people
who were damn scared, felt as if he had instead ________ a spirit, as they felt the stronger presence of
something new in the room.

A. convened
B. generated
C. excogitated
D. evoked
E. fused

41. A guy used to bully all his shy and quiet natured classmates, but when he received a/an _____________
comeback from one of the guys who was actually very quiet, he realised that his way of judging people
to bully them may be wrong and strangely he still didn’t realise that bullying others wasn’t morally
right.

A. insipid
B. muffled
C. nebulous
D. diplomatic
E. acerbic

42. The professor seemed to be quite friendly during his PhD interview, which lured a candidate to choose
him as his PhD advisor, little did he know that professor followed a vastly more _______ style, a fact
which he learned to his chagrin within the first few days of his work with him.

A. draconian
B. amiable
C. vacuous
D. bothersome

Page 30 | Text Completion | www.collegepond.com


E. bewildered

43. Climate change is considered by President Trump as a hoax spread by the Chinese, but he should once
see the steady decrease in the _____ of Antarctica and Greenland. Only then he may realise that melting
ice will raise sea levels all over the world and all the coastal cities will be totally destroyed.

A. Floes
B. accidents
C. cads
D. dirges
E. deceptions

44. Rohit Shetty, Salman Khan and super-hero fantasy genre movies have one thing in common; all the
villains and goons, around 15 in number at the least will fly fancily and soar high in the sky with just 1
punch of the hero, still people spend money to watch such ___________ action, arching far beyond reality.

A. tit-for-tat
B. Fable type
C. Plight of
D. insinuating
E. hyperbolic

45. MS Dhoni, the captain Cool is known for his cool composure as well as aggressive attacking captaincy,
thereby he is a classic example of a person with ___________technique.

A. morbid
B. symbolic
C. eloquent
D. contrapuntal
E. orchestrated

46. Engineers around the world suggested that there isn’t an alternative but to demolish the bridge
completely and reconstruct a new one. However, one smart Engineer suggested to use the flying
________, as the bridge had started to sag and needed additional support.

A. ballast
B. characteristic
C. buttress
D. seal
E. hindrance

47. The businessman tried hard to not invest in the new-age extremely costly technology, but his time was
about to end at the company as he approached a dying age, and his son was supposed to take over. He
gave his _________ to the risky investment in the new technology as he felt it may work after all, when his
son explained him all the pros and cons.

A. assent
B. protest
C. descent
D. conquest
E. asseveration

48. Jon Snow, did not fear death and to protect the realm from the White Walkers, he did not care for self
and fought even when wildlings and white walkers threatened to _________ him limb from limb.

A. amass
B. rend

Page 31 | Text Completion | www.collegepond.com


C. evince
D. embellish
E. garland

49. Owing to the toughened H1B visa restrictions and a/an_________ job market, many MS graduates are
continuing PhD studies rather than competing for the very few available jobs.

A. myopic
B. ceremonial
C. fissile
D. anaemic
E. bungled

50. The movie Anand was more than just a hit movie- it features in the list of all time favourite movies as it
was a cultural __________ wherein they showed human relationship in a different light and that liveliness
could conquer even the non-curable diseases.

A. rudder
B. fortify
C. stanchion
D. crux
E. touchstone

Page 32 | Text Completion | www.collegepond.com


Text Completion 1 blank drill solutions

1. Apportioned. The word “whereas” creates a contrast, so you need Australian fans to be evenly split
between their many sports. Measured is close, but doesn’t quite divide things up the way you need it to.
Apportioned means “divided and allocated.”

2. Delusion. The second half of the sentence tells you that it would take an effort for a person to understand
about the actual location and shape of Earth. In other words, it’s more or less or impossible for them at
current stage to explain ignorant people. Frenzy is too negative and extreme. Delusion, meaning “a belief
that is maintained despite being contradicted by reality,” is the best fit.

3. Preposterous. The blank must agree with the clue that the possibility is remote.

4. Enduring. The blank must agree with destruction that has prevailed through years due to nuclear bombing
Thus enduring is correct. Note that incorrect choice ephemeral is exactly the opposite of the intended
meaning.

5. Superstitious. The pivot word because indicates that the blank will agree with the (interesting but
meaningless) pattern that changing the name can bring success. Don’t fall for traps like orthodox (which
would only be attractive if you were using outside information).

6. Circle. The clue “in turn” leads to the correct answer. Incorrect answers irony and satire add an idea —
humor — that was not indicated by the sentence.

7. Abreast of. The word apart from gives you a clue that even if you have knowledge, you are required to
update yourself. To keep abreast of a topic is to remain current.

8. Inundated. Upon reading this sentence, you might have filled in something like “went to or migrated” in the
blank. Process of elimination works well here — annihilated, ostracized, shunned and rebuked, all add
ideas not indicated by the sentence. Only inundated (flooded, overwhelmed) makes sense.

9. Coda. You need a word that means something like a bad ending. A coda is an ending that sums up what
came before, which, in this case, was pretty bad.

10. Inured to. Casting a blind eye to suffering means that these people ignore or pretend not to see the
suffering — they are hardened and desensitized as a result of seeing so much of it. Trap answer spared
would mean that the people described are no longer personally subject to the suffering. You don’t know
that, of course. You only have the clue that the people cast a blind eye, or ignore suffering. Because they
ignore suffering, they also wouldn’t be maudlin or effusive, which are about expressing strong feelings.

11. Coalesced. You need a word that means came together. Coalesced matches perfectly. Agglomerated is a bit
of a trap — to agglomerate is to collect or form into a mass or group. Since the people were already in a
group, the word is inappropriate (agglomerate is also used more for things than for people).

12. Anachronisms. The director inserted some things in the wrong time period, or anachronisms.

13. Enigma. Since no one has been able to decode the reason, you need a word that means “mystery.”

14. Inform. The writer’s breakups memories influence his work. You need a similar word for how losing his
parents influence his story writing. Inform — to give substance, character, or distinction to — can be used
in this way.

15. A risible. The candidate went for the interview and is speaking German in the wrong room and confusing
everyone? Hilarious! Risible means “laughable” and is the only choice that makes any sense here. Note that
arch can mean “mischievous, playful, roguish,” but is not appropriate here because the candidate did this
“accidentally.”

Page 33 | Text Completion | www.collegepond.com


16. Facile. The engineer’s suggestion is rather childish; you also have the word “naively” as a clue. Facile
matches well, and has the sense of “superficial; so simple it’s actually stupid.” Note that intricate often
describes very complicated or detailed — this choice does not match the clue “naively.”

17. Convivial. Extroverts are sociable, outgoing, etc. Only convivial matches.

18. Novel. The correct answer must match the clue “new” and the structure that contrasts the blank with “time
honored.” Upheaval is tempting but not the correct answer.

19. Hysterics. The blank must contrast “maintain her composure.” Note that while, in casual speech, many
people use hysterical to mean “funny,” that is a metaphorical usage — hysterical actually relates to being
very upset.

20. Artifices. The pivot means the answer must be in contrast to the clue, “represent real fighting which gives
authenticity.”

21. Pedantry. The correct answer should parallel “dogmatic” and match the keywords “does not inspire”. Only
pedantry (being arrogant in relation to learning) fits the idea of an uninspiring professor.

22. Minutiae. The correct answer must reflect the clue, “petty,” and oppose the idea of “worthy targets.” He
paid so much attention to details that he ignored the big picture.

23. Enhances. You simply need a word here that means “helps.” Don’t fall for guarantees, which is too extreme.

24. Parlous. The correct answer must match “dangerous.” Note: suicidal is too extreme and not indicated by the
sentence.

25. Trundled. The clue “slowly” points to the correct answer — only trundled can describe a slow movement.
Careened describes a fast, out-of-control movement, and tarried describes a person lingering.

26. Didactic. The word didactic can mean “morally instructive” or just “instructive” — this book is a didactic
work.

27. Disinterested. You need a word that means unbiased (a good quality for a match referee). Disinterested
does not mean uninterested — rather, it means not having a “personal interest” in the topic, and therefore
being unbiased.

28. Risible. The clue phrase here is “exaggerated hilarity,” letting you know that he was amusing even though
people would expect him to be sad. Risible, which means “laughable,” is the most appropriate.

29. Belies. You are searching for a contrast word, specifically one indicating that there is more to the overnight
success than meets the eye. Belies, one meaning of which is “to be at odds with,” is most appropriate here.

30. Premature. The clues are “initiative.” You don’t know if these Buddhists were violent (gory) or dogmatic —
avoid opinions or outside knowledge.

31. Gleaned. Gleaned means “extracted from various sources.” Perused is the act of reading — you can peruse a
book, but you cannot peruse a discovery from a book.

32. Chide. The adverb gently is important here, because it’s impossible to “gently” despise someone or “gently”
pillory them (pillory means “attack or ridicule publicly”).

33. Loaf. If teenagers haven’t been doing anything useful with their time, you need a word that means do
nothing useful with your time. Flirt is something teenagers do, but it isn’t useless by definition (it can be
quite fun!). Contemplating can also be useful, as it involves thinking. The correct answer is loaf. Note that
the overall tone of this sentence is a bit sarcastic.

Page 34 | Text Completion | www.collegepond.com


34. Spectre. For this sentence, it’s important that you know that nuclear holocaust never actually occurred, so it
was only a threat that was a daily part of people’s lives. A secondary meaning of the word spectre (which
can also mean “ghost”) is “something widely feared as a dangerous occurrence.” All the other words have a
kind of categorical relevance, but they don’t fit the sentence.

35. Rail against. This sentence shows you that the courts have presented a proper fact as to why the practice
doesn’t make sense. Both skitter around and pass over would imply the courts fail to find examples or facts,
and conspire with is the opposite of what you want. Rail means “complain or protest strongly,” and is
correct here.

36. Puffery. While babble is rubbish talk, and confabulation is invented talk, only puffery means “exaggerated
or false praise,” which is exactly what you want here.

37. Decry. Decry is “to speak disparagingly” and parallels the clue, “curse.” None of the other choices are
justified by the meaning of the text.

38. Robust. Since the bacteria and virus may cause the disease again, on stopping of treatment, so they are the
ones which are the hardest to kill. So blank should have a word meaning drug resistant. Robust suits here.

39. Prodigy. Son of Paresh Bambani was expected to be a spoiled brat, like the sons of various rich
personalities. But he wasn’t a stereotype, but a child who disproved the stereotype, so we want something
that means child who is preternaturally intelligent. Prodigy suits here.

40. Evoked. Warren was actually trying to shoo the spirit away but the two other scared people felt as if he
instead called the evil spirit in the room. Evoke and convened are closer in meanings. But evoked is more
appropriate as it means summoned and because convened is usually used to call a meeting.

41. Acerbic. The used to bully all quiet people considering them to be weaker. However, one of them gave a
befitting reply. So we need a word which means sharp and bitter comeback. Acerbic suits here.

42. Draconian. Lured means that he fell in some kind of trap and attraction. Little did he know - also helps us
understand that the student’s belief and judgement of professor was not correct. So we need a word that is
opposite of friendly. The new student also experiences chagrin meaning mental distress. Draconian which
means harsh, strict or severe is the best fit.

43. Floes. We need a synonym of melting ice - glaciers. Floes which means the same is the correct answer.
Dirges is a trap answer that is related to “lamented”.

44. Hyperbolic. The answer must match the clues “fly fancily” and “arching far beyond reality.” Hyperbole is the
correct answer

45. Contrapuntal. We want a word which means having two components. Contrapuntal is the right word.

46. Buttress. Last phrase indicates bridge needed additional support. You don’t need to memorise jargons but
you should know the metaphorical meaning of buttress which is to support.

47. Assent. It may work after all. The last line shows that old man may have finally developed faith in
purchasing the new expensive technology. So we need something like agreement of his for his son’s
decision.

48. Rend. Rend means “to tear apart.” This question relies entirely on vocabulary.

49. Anemic as a metaphor means “weak or lacking vitality.” Myopia, or “nearsightedness,” may be a tricky
choice and it also makes a good metaphor, but describes people and their decisions.

50. Touchstone. You need a clue that matches more than a hit movie (and goes with the idea of having meaning
for a whole generation of people). Touchstone matches perfectly.

Page 35 | Text Completion | www.collegepond.com


Our University
Admits

022 - 2610 3135 | 022 - 2619 5832 Collegepond


info@collegepond.com www.collegepond.com
Text Completion 2 blanks drill

1. Peculiarly enough, Sachin Tendulkar has been often (i) ______ as the best batsman in the history of cricket,
and often (ii) ______ as a man lacking the education.

Blank(i) Blank(ii)
A crowned D demonized
B vilified E dismissed
C chastised F venerated

2. Graduate admissions in a top school work through professor’s recommendations. Therefore, though the
graduate school does not (i) _________ the recommendation of its faculty, the graduate school still has the
power to overwrite professor’s recommendation and just the professor’s recommendation regarding
affirmative selection of candidate could not remain (ii) _________ as the GRE and TOEFL scores should also
meet the minimum requirements set by the graduate school.

Blank(i) Blank(ii)
A defend D unavailed
B ruminate E unchallenged
C Undermine F averred

3. Anish Mehrotra, the number one ranked fashion designer of India, known for his whimsical but acerbic wit,
commented, in his usual, simultaneously (i) _________ and (ii) _________ manner, that Ranveer Seth should
model for women instead of men, looking at the way he has been dressing lately in public.

Blank(i) Blank(ii)
A scandalous D solicitous
B humorous E hysterical
C grave F caustic

4. Anna Hazare, well-acclaimed social activist, was not the only (i) _________ of the long-proposed Lokpal bill,
but he was the (ii) _________ who finally got the bill onto the legislative agenda in the House after several
Satyagrahas and Anshans.

Blank(i) Blank(ii)
A defender D catalyst
B disparager E avaricious
C advocate F lackey
5. Mark Zuckerberg, was not the one to (i) _________ his success, so his parents were shocked when they finally
discovered that their (ii) _________ son had co-founded a social networking site, which was such a hit that he
became the youngest billionaire.

Blank(i) Blank(ii)
A Delineate D prodigal
B camouflage E taciturn
C trumpet F profligate

6. Their love didn’t need any sanctioning and this was proved by them by wedding each other even when they
lost their wedding ring. But on their 50th anniversary, the beautiful woman who had now become old was
(i) _________ to have found her ring that she had lost in the (ii) _________ waters of the Mississippi River.

Page 36 | Text Completion | www.collegepond.com


Blank(i) Blank(ii)
A befuddled D turbid
B dumbfounded E turgid
C devitalized F tepid

7. Though the novel-adopted field gained a large amount at the Box Office, but the novel was (i) _________ (ii)
_________ by critics; not a single reviewer had a single positive thing to say about it.

Blank(i) Blank(ii)
A cautiously D extolled
B fraudulently E panned
C roundly F revered

8. The movie Lunchbox greatly depicts that one doesn’t need physical presence of another person to portray
our love for them. Even though the actor Irfan Khan hadn’t met Nimrat Kaur, they felt they were of one
mind, each (i) _________ anticipating the contents of the other’s letters-the only communication form they
had to talk with each other; never had two persons in love with each other without knowing them, been
more (ii) _________.

Blank(i) Blank(ii)
A presciently D propitious
B anachronistically E astute
C menacingly F simpatico

9. Tomato is often (i) _________ referred to as a vegetable; (ii) _________ to such people, it is actually a fruit.

Blank(i) Blank(ii)
A archaically D unbeknownst
B erroneously E despicable
C dexterously F consanguineous

10. Bungee jumping isn’t a sport that can be practiced by weak-hearted people. The (i) _________ 20-year old
man approached the edge of the diving board; (ii) _________ the jump in the first place, he found the prospect
of jumping into it from such a height even more frightening.

Blank(i) Blank(ii)
A tremulous D enchanted by
B coltish E repulsed by
C clumsy F chary of

11. In the movie Nayak, Anil Kapoor, ruled the state of Maharashtra, as the Chief Minister for only a day;
despite its (i) _________ reign, it was at the time (ii) _________ as a sign of the emergence of a powerful working
middle class and evolution of good governance and prosperity for poor people.

Blank(i) Blank(ii)
A equivocal D discounted
B ephemeral E renounced
C despotic F heralded

Page 37 | Text Completion | www.collegepond.com


12. People believe the bungalow at the end of the street is haunted and several paranormal activity is observed
in the late hours of the night near the bungalow. This has resulted in people living in nearby bungalows
vacating their houses. As a result the housing property has now fallen into (i)_____________ and appears
positively (ii) _____________ by vermin.

Blank(i) Blank(ii)
A cynicism D overrun
B desuetude E outclass
C debauchment F inhibited

13. Some people are born with the skills of their parents. So, when Mr. Mehra’s teenager son, who was pleasant
enough, came for the board meeting, Board of Directors thought that the (i)______ junior Mehra was hardly
(ii) _________ enough to understand high level business strategies and bargain and devise new plans to
maximise company’s profit in the new category of products.

Blank(i) Blank(ii)
A puissant D egoistic
B fidgety E nonchalant
C green F artful

14. The beautiful gorgeous looking girl, who people thought would marry some muscle-hunk man, actually fell
for a thin, not-so good looking guy as the guy had such a pleasingly (i) __________ personality that she wasn’t
bothered about the (ii) _________ in his past or his malnutritioned looks.

Blank(i) Blank(ii)
A sanguine D peccadilloes
B imperious E insurgencies
C salutary F attainments

15. After losing several million dollars due to a bad marketing gimmick and competition from other brands, the
boss decided not to further hire any (i) _________ young employees due to the low budget and also (ii)
_________ the marketing team for making a serious mistake that cost the company thousands of dollars.

Blank(i) Blank(ii)
A trite D dismiss
B ginger E castigate
C verdant F enthral

16. The audition for hosting of a local award function was attended by several confident personalities,
however, there was also one new entry, who spoke with an (i)___________ , that (ii) ________ audiences; an
irregular reluctance and understatement compared to other performances given by the ostentatious peers
in the auditioning event.

Blank(i) Blank(ii)
A overwhelming D captivated
B awkward lisp E delighted
C engaging insecurity F fervour repelled

Page 38 | Text Completion | www.collegepond.com


17. Ironically, the son of a rich multi-millionaire father, who used to send forwarded messages about working
hard, saving money and who condemned personal (i) _________ was (ii) _________ to the point of bankruptcy.

Blank(i) Blank(ii)
A Hiatus D spendthrift
B dissipation E primal
C rectitude F austere

18. In the modern era, where people believe the captain plays the most important role in winning a match, MS
Dhoni, the most successful captain of the Indian Cricket team, appeals the audience not only because of the
(i) _________ displayed in his captaincy, but he also offers (ii) _________ performance through his high strike-
rate fierce batting where the rest of the players just struggle to make a decent score.

Blank(i) Blank(ii)
A doctrine D trenchant
B eccentricity E bland
C inheritance F inexplicable

19. When India fought for its freedom against the British, the revolution couldn’t be just categorised against the
British. There was also a fight for who would rule India after it became free from the clutches of the British.
So, it can be said that transitioning of power during revolution is really (i) _________ because the power of
common men in the revolution leads to transfer of power inexorably from the more stable (oppressive)
forces to less temperate ones. However, after a period of 20 years or so, the excesses (ii) _________ and once
the new government stabilised, a more peaceful period started.

Blank(i) Blank(ii)
A Brisk D metastasized
B berserk E grew
C mild F receded

20. MS Dhoni also known as the Captain Cool, is known for his (i) _________ even under the extreme tense final
situations of the match. But, in the middle overs, he is often (ii) _________ with his bowlers, who never knew
when they might be subjected to one of his quirky comments or tirades for not bowling well.

Blank(i) Blank(ii)
A undismayed D fascist
B sangfroid E mercurial
C iniquity F pompous

21. Back in the 80s and 90s, when the gangsters were more powerful than the police and the government, it
was apparent that during any negotiation between the two gangster teams, any appearance of (i) _________
between the teams was deceitful or, at best, a superficial adherence to certain (ii) _________, which was
actually nothing but a dirty politics game, metaphorically, with one hand for handshake and other with a
knife ready to stab.

Blank(i) Blank(ii)
A chicanery D mores
B consolation E politics
C amity F plaudits

Page 39 | Text Completion | www.collegepond.com


22. Many people in India became furious when they found out about the Saint Asaram Bapu’s (i) _________ late in
his life, which also (ii) _________ his reputation, others also felt that his dissolution should make us aware
that all these Saints are actually frauds, who just claim of not being attracted by females, but in reality
libertinous tales can be written for each one of them.

Blank(i) Blank(ii)
A blemish D denigrated from
B debauchment E aggrandized
C degradation F bedizened

23. The Engineering students of the India have given rise to a famous troll or “meme”, where their plan of a trip
to Goa is more of a (i) _________ vision than a concrete proposal; like many other similar (ii) _________ ideas
such as starting and working for their own company, successfully convincing brown parents for live-in
relationships, it is unlikely to ever come to fruition.

Blank(i) Blank(ii)
A Dionysian D quixotic
B protean E Bohemian
C pre-emptive F pragmatic

24. The most popular show of all time, the fantasy show by HBO, “Game of Thrones”, a (i) ________ account of a
(ii) _________ life, is so gripping that people are getting emotionally attached with one or the another
character from the show; though show isn’t appropriate for young kids and teenagers, it still enjoys All time
highest rated show popularity.

Blank(i) Blank(ii)
A gaucherie D jejune
B Inveterate E stonewalled
C no-holds-barred F storied 25. The reservation system in the education was
made to help SC, ST, OBC students get higher education, because even after independence, these people
were treated as slaves and weren’t allowed the right to education. So, when an erudite engineer but selfish
businessman proposed a (i) _________ suggestion that reserved category backward class people be shuffled
off to a/an(ii) _________ organization where they could work with underprivileged and backward class
people with similar status as theirs and earn little money throughout their life, the other successful people
from the backward classes got furious over the man’s selfish motives.

Blank(i) Blank(ii)
A regressive D a hierarchical
B bucolic E an auxiliary
C priggish F a provocative

26. Govinda, although once considered to be the “Hero Number 1” of Bollywood, (i) ___________ actor, lost his
charm and popularity when he joined politics. The movie critics now are unable to (ii) ________ the interest
of struggling and aspiring actors, in the actor who was once considered to be the legendary and the most
popular actor of Bollywood, due to his bad political career.

Blank(i) Blank(ii)
A a furtive D whet
B a deft E endorse
C an effeminate F amass

Page 40 | Text Completion | www.collegepond.com


27. Many multinational companies buy smaller companies as these small companies are legally shielded from
tough laws of prosecution when hiring outside firms for clerical labour work; this clearly shows that
smaller companies (i)____________ with the MNCs, (ii) _____________the larger firm’s plans to not face legal
prosecution yet do illegal work of outsourcing and thereby maximise their own profits.

Blank(i) Blank(ii)
A at loggerheads D conspiring
B pensive E abetting
C in cahoots F stymying

28. After Donald Trump was elected as the President of USA in 2016 Presidential Elections, he famously (i)
_________ approval of the “Protect American Jobs” Act, whereby he targeted H1B visas, so the hiring rate of
big tech giants reduced due to the fear of (ii)__________ power at the hands of the President and his executive
team which may change the hiring rules for internationals as and when they like.

Blank(i) Blank(ii)
A scorned D adduced
B heralded E represented
C ratified F appropriated

29. Local people generally fear police. Frightened by the police’s purportedly (i)_________ inquisitions, a local
businessman shut all his ongoing projects and (ii)__________ with his family to another city..

Blank(i) Blank(ii)
A dubious D immigrated
B benign E absconded
C Racy F gratified

30. The relationship between Donald Trump and Narendra Modi has gone from positively (i) _________ to chilly
at best, not least because the recent H1B visa rule changes threatens to (ii) _________ the mutual trust that
has been held on both sides for years as both the nations benefited from the program, US with skilled
labour and India with diverse opportunities for its citizens.

Blank(i) Blank(ii)
A Frugal D deteriorate
B congenial E erode
C deferential F fortify

31. It has been a topic of debate that which drugs are more relaxing, the ones that induce lassitude and torpor,
or the ones that incite (i) _________ or (ii) _________.

Blank(i) Blank(ii)
A convulsions D grievance
B ennui E retching
C squabble F drowsiness

Page 41 | Text Completion | www.collegepond.com


32. Even after women and Dalits got equal rights as the other citizens as per law, (i)____________ did not stop and
these backward people got equal respect in the society only after 1990s in India and so it could be said that
they were granted (ii) _____________ after IT revolution and globalization, where news channels spread such
information to masses and public slavery of dalits and women thus began to decrease.

Blank(i) Blank(ii)
A disenfranchisement D hegemony
B jingoism E exoneration
C plutocracy F suffrage

33. The life of a comedian or joker isn’t as easy as it seems. Many of them experience subtle psychological
damage each time they (i) __________ their true emotions and put on a smile for the customer. Unfortunately,
such efforts are (ii) ___________ of the job, and they can’t risk to disappoint their audience.

Blank(i) Blank(ii)
A dissimulate D the pinnacle
B imitate E the conceit
C mask F part and parcel

34. Indian workers prefer government jobs even though it’s easier to earn higher salaries in private companies,
because (i) ________ as ample vacation time and excellent insurance persuade people to (ii) ______.

Blank(i) Blank(ii)
A perquisites D settle
B remittance E abide
C deterrent F abscond

35. While many people like the oratory skills of late Shri Balasaheb Thackeray, also known as the Hindu
Hruday Samrat, but there were some who condemned it as (i) ________ for their religion, and therefore can
be categorised as (ii) ________.

Blank(i) Blank(ii)
A Vulgar D interlocutors
B blunt E detractors
C courteous F Companions

36. Researchers are developing sophisticated and (i)______ Artificial Intelligent machines to lighten the burden
and make the tasks of humans easier. These intelligent machines can be employed to help (ii) ______ even
decisions that are not primarily related to its main primary purpose after gaining feedback from its human
boss.

Blank(i) Blank(ii)
A Liberal D extrapolate
B populist E reproduce
C versatile F explicate

37. A (i) __________ device, developed as the structural model of psyche, was useful in inciting discovery and was
not a mere attempt to (ii) _______ physical relationships among parts of human brain.

Page 42 | Text Completion | www.collegepond.com


Blank(i) Blank(ii)
A a heuristic D domineer
B an emblematic E represent
C a hermeneutic F palliate

38. The show Gutur Gu, made lives tough for themselves by creating a virtue of the (i) __________ limits, depicting
people who just used gestures to make audience laugh and even though the actors acted so (ii) ________ , the
show was able to make the audience laugh even when the medium lacked the voice.

Blank(i) Blank(ii)
A artistic D avant-garde
B commercial E diffident
C technical F reluctant

39. Things were going fine at the dinner party, unless the drunken businessman who had threw the party
started joking about the faults of her wives, which eventually grew too (i) ___________, becoming so bitter,
ironic and personal that the audience found his talk very (ii)__________.

Blank(i) Blank(ii)
A distraught D disquieting
B mordant E antagonistic
C Pettish F buoyant

40. Not only is Jeffrey Archer good in writing thriller novels, his early efforts at romantic, drama and
mainstream genre (i)________ his ability to represent the more (ii)__________ world through his writing.

Blank(i) Blank(ii)
A attest to D ebullient
B belie E quotidian
C gainsay F quotidian

41. Hairloss and balding, one of the most common phenomenon occurring particularly in males, from
generations, sometimes makes us wonder that why haven’t the scientists and researchers been able to
figure out the actual reason of balding and thus suitably devise a treatment for the same. The fact that we
are still so ignorant about a topic so (i)_________ to our daily lives is actually (ii)_________.

Blank(i) Blank(ii)
A mysterious D deeply humbling
B pertinent E totally blatant
C obscure F fully impenetrable

42. (i) __________ comprehension of the character of rice and mango fruit is impeded not only by labyrinthine
complexities of soil and fertiliser, but fluctuations in the environmental conditions every year, also play a
major role which make the most reliable crop produce subject to (ii) ___________.

Blank(i) Blank(ii)
A An exhaustive D efficient taxonomy
B A fruitless E remarkable variance
C An irredeemable F mitigating circumstances

Page 43 | Text Completion | www.collegepond.com


43. Government employees usually treat all their customers or clients with (i) ____________ and this shows a
more general lack of respect for his office; he seems to treat the whole thing as if it were some grand (ii)
___________.

Blank(i) Blank(ii)
A casual nonchalance D gesture
B profound meticulousness E farce
C idle envy F diatribe

44. (i)_____________initially the business proposal had little impact, but it was (ii) __________ subsequently by the
head tycoon, known for his wittiness and quick accurate decisions which provided support to secure
funding from investors.

Blank(i) Blank(ii)
A Justifiably D rendered moot
B Unbelievably E given teeth
C Initially F kept at bay

45. John Goodenough, researcher who gave the world lithium-ion batteries, has come up with a new
technology, and has performed experiments to prove the (i) _________ of the theory, but an empirical relation
can’t be set up unless the data is (ii)__________ with other experimental results carried around the world’s
different labs and thus the technology cannot be claimed to be a successful invention as of now.

Blank(i) Blank(ii)
A rationality D consistent with
B veracity E founded on
C directness F antithetical to

Page 44 | Text Completion | www.collegepond.com


Text Completion 2 blanks drill solutions

1. Crowned, dismissed. “Peculiarly enough” is the clue that indicates that the blanks oppose each other, and
best batsman is the clue that tells you that the first blank must be positive, so the second blank must be
negative. Crowned is the only positive word choice for the first blank. In the second blank, demonized is
much too strong an attitude to direct towards someone for lacking education; dismissed is a more
appropriate word.

2. Undermine, unchallenged. Therefore, though …. tells you that the two parts of the sentence will oppose
each other. The second suggests that the graduate school believes the professor’s recommendation couldn’t
be allowed to stand as it is, unless the student also has sufficient GRE TOEFL scores, so they could not go
unchallenged. If the graduate school challenges a faculty member, however, this tends to subvert or
undermine the faculty member’s recommendation, which is what you need for the first blank. Note that, if
you picked defend for the first blank, you would need a word like undefended in the second blank — none
of the choices for the second blank match undefended.

3. Humorous, caustic. The clue “usual” means that the two blanks must match “whimsical but acerbic” — and
in order

4. advocate, catalyst. You know that Anna Hazare finally got the bill onto the legislative agenda, so he is
definitely an advocate, or proponent, of the legislation. A catalyst — a term adopted from chemistry — is
someone who creates a change.

5. Trumpet, taciturn. You know that Zuckerberg is successful, but it takes his parents a long time to “discover”
his accomplishments. Thus, Mark does not brag — or even report the facts! To trumpet is to “talk loudly or
report something to everyone.” In the second blank, only taciturn (quiet) works. Don’t be fooled by
prodigal, which occurs in the expression “prodigal son,” but actually means “wasteful.” (The original
prodigal son from the Bible ungratefully wasted his inheritance.)

6. Dumbfounded, turbid. In the first blank, you need a word like amazed. Dumbfounded is the only match.
Turbid which is related to turbulent, is the right word to describe choppy waters in which something could
easily be lost. (Turgid means “stiff” and tepid means “lukewarm.”)

7. Roundly, panned. Wow, what a terrible novel! Not a single reviewer could think of one nice thing to say?
You need two words that mean something like “unanimously criticized” or “100% disliked.” Roundly means
“emphatically,” or “so thoroughly as to leave no doubt.” Panned means “reviewed negatively” and is almost
always used to refer to plays, movies, etc., so the word is a perfect match here.

8. Presciently, simpatico. Presciently means “seeing into the future,” or “as though seeing into the future.”
Simpatico matches the idea of being “of one mind.”

9. Erroneously, unbeknownst. Since the tomato is not really a vegetable (it is a fruit), erroneously
(incorrectly) matches in the first blank. Unbeknownst to means “unknown by.”

10. Tremulous, chary of. All of the clues in the sentence are about being scared, so tremulous matches best in
the first blank. You know that the child finds diving “even more frightening” than jumping in general, so
chary of (suspicious or slightly scared of), a less extreme version of frightening, fits in the second blank.

Page 45 | Text Completion | www.collegepond.com


11. Ephemeral, heralded. Since the government lasted only about 1 day, it was ephemeral, or “short-lived.”
Despite this, it was regarded, or celebrated, as the emergence of a powerful working middle class: only
heralded matches.

12. Desuetude, overrun. Desuetude means “disuse,” although the word is usually used in a legal context (a law
that has not been enforced in a very long time may fall into desuetude and in some legal codes be no longer
valid). Overrun matches the idea of having many vermin (rats and other undesirable animals). Don’t fall for
trap answer hampered — the house couldn’t be hampered because it wasn’t trying to accomplish anything.

13. green, artful. You know from the sentence that Mr. Mehra’s son is “pleasant” but that he may not be old
enough to understand company’s business strategies. In the first blank, green means “young and
inexperienced” and in the second, artful means “crafty, cunning, shrewd” — qualities important for
bargaining.

14. Sanguine, peccadilloes. You know that the man’s personality is pleasing — so pleasing that it’s hard to be
bothered by something in his past. So, the first word should match pleasing and the second should be
something negative that others might overlook. Sanguine means “cheerful and optimistic.” Peccadilloes are
minor faults or sins — small enough that they might be overlooked if the person has other virtues.

15. Verdant, castigate. Since the experienced employees also made such a big blunder, the boss doesn’t want to
risk it with new young inexperienced employees and also he is low on budget to hire new employees.
Verdant is the perfect word which means either “green and lush” (like a forest), or “new and
inexperienced” (the use of “green” here is as a metaphor, as in, “She’s still green at her job.”) Since the
mistake was so expensive, it makes sense that the current team would be castigated, or severely criticized.
(He wouldn’t be deposed — that word is for kings, dictators, etc., forced out of power.)

16. Engaging insecurity, captivated. The second blank is easier; given the structure, it must compliment the
clue “ostentatious” in meaning and spin. The first blank must parallel “irregular reluctance and
understatement” and also have a positive spin.

17. Dissipation, spendthrift. The clue that can be used to find out answer is the word “Ironically”. The son who
used to advise people to save money and condemn people for the same was actually himself spending in a
profligate manner. Both blanks should be negative, similar in meaning, and matching the idea of
“bankruptcy.” Dissipation and spendthrift are both related to wastefulness or irresponsible living.

18. Eccentricity, trenchant. The first blank is tricky. The most successful captain helps us understand that there
is something peculiar about Dhoni’s captaincy. Thus, you need a word that means something like
“uniqueness.” In the second sentence, Trenchant conveys that Dhoni’s work is serious and vigorous in
expression. “Fierce batting” help us further by acting as a clue.

19. Berserk, receded. Transferring power to “less temperate” forces would cause chaos. You also have the clue
at the end — a “more peaceful” period comes later. So you need a pretty extreme opposite of “peaceful” in
the first blank. Only berserk works. In the second sentence, excesses recede (reduce, pull back), allowing a
more peaceful phase.

20. Sangfroid, mercurial. A tricky structure — while the second blank is opposite of the clue “equilibrium”
(because of the “but”), the first blank should actually have the same meaning as equilibrium.

Page 46 | Text Completion | www.collegepond.com


21. Amity, mores. The first blank comes after “appearance of,” which indicates fake behavior. Deceitful is the
word that backs up that meaning. You need something positive — the parties are showing false amity, or
friendliness, out of a “superficial” adherence to customs or rules — mores. Don’t be attracted by the word
politics as it is again later present in the blank, understand the context of the blank.

22. Debauchment, denigrated from. Our clue is Asharam Bapu’s “dissolution” — he turned to promiscuity, rape,
etc and just let his life fall apart. Libertinous has a similar meaning, as does the correct word debauchment.
The 2nd blank should be something negative resulting from degrading activities of the saint.

23. Protean, quixotic. In the first blank, you need an idea that contrasts with “concrete proposal.” Protean
matches nicely. For the second blank, you need a word that also represents the idea of not being concrete,
as well as the idea of being unlikely to ever come to fruition. Quixotic matches perfectly.

24. No-holds-barred, storied. You are told that the show is so gripping, that people are emotionally getting
attached to one or other character, but it is not appropriate for young kids and teenagers. So, it’s exciting —
and probably full of sex, violence, power, politics or all of them. No-holds-barred and storied are the only
choices that could even remotely match — surely the biography isn’t pretending to be awkward or
unsophisticated (gaucherie) or unwilling to change (inveterate), and the “life” isn’t naive (jejune) or stalled
(stonewalled).

25. Regressive, auxiliary. The question takes a lot of turns. However the main motive is explained through
businessman’s selfish motives that though he is from backward class, he doesn’t want other backward
people to progress and become rich. That’s why his suggestion wouldn’t be a positive and modern one but
would be to limit others from backward class. In the second blank, you want something that reflects the
idea that these people should be grouped with and work with backward class people only. Auxiliary has
this sense of “secondary” or “on the side.”

26. A deft, whet. Due to the “although,” you can infer that the first blank must be something positive — only
deft works. In the second blank, whet means “stimulate.” Tout is a bit of a trap — the movie critics could
tout Govinda as a fun thing to learn about, but he can’t tout someone’s interest.

27. In cahoots, abetting. MNCs trickily find a way to maximise profits by outsourcing their tasks. But they don’t
directly do it themselves fearing chances of prosecution. So, they purchase smaller companies which help
them achieve the task. So, smaller companies work with the larger ones in doing wrong - it is in cahoots. In
the second blank, you need a word that means “complicit in” or “helping (in a bad way).” Only abetting
works.

28. Heralded, appropriated. The phrase “famously” gives you a clue that approval was highly publicized, or
heralded. For the second blank, you aren’t given much of a clue about what the meaning should be. When
this happens, only one answer choice will make sense (and usually that correct choice won’t add much to
the sentence), while the other, incorrect answer choices will drastically change the meaning. Here,
appropriated, which means “allocated” or “assigned,” fits nicely and doesn’t add any unintended meaning.
Neither adduced, which means “cited as evidence,” nor represented is fitting.

29. Benign, absconded. The word purport, which means to “pretend to be”” The sentence implies that the
police is pretending to be harmless, or benign. For the second blank, you are looking for a word that means
to “leave hurriedly”; absconded fits. Notice that immigrated, though close in meaning, isn’t quite right
because it means to “arrive at” a country or region rather than to “leave from” it.

Page 47 | Text Completion | www.collegepond.com


30. Congenial, erode. “chilly at best,” helps us understand that relationship was warm or cordial at first.
Knowing that the relationship has devolved allows you to turn to the second blank and determine that
there has been an undermining, or an eroding, of “mutual trust.”

31. Convulsions, retching. The two blanks contrast “lassitude and torpor” because the first line says it has been
a topic of debate. Additionally, the word incite indicates that the blanks will contain words that are active
rather than passive.

32. Disenfranchisement, suffrage. The sentence gives you two examples involving the restriction of voting
rights. Disenfranchisement is the deprivation of the right to vote, which fits best with the topic. Similarly,
suffrage is the right to vote, and is the correct word for the second blank.

33. Mask, part and parcel. Comedians have to make people laugh. So no one would like them if they themselves
are sad. So even if they are sad they have to put a smile and hide their true emotions. Mask means hide. For
the second blank, you need something like a requirement. Part and parcel, meaning “an essential piece of
something,” is the correct phrase.

34. Perquisites, abide. For the first word you want something like “non-salary rewards,” and the only word that
means that is perquisites (often informally abbreviated as “perks”). For the second word you want
something like “stay,” and only abide can have that meaning. (Abide more commonly means “tolerate” or
“wait for.”)

35. Vulgar, detractors. Many people liked oratory skills of Mr Thackeray. But some condemned it so we need a
negative word. Vulgar is the best choice as people who condemned it were those against whose religion
Thackeray would speak. Second blank detractors fits the best.

36. Versatile, explicate. New AI machines are self-learning and therefore can learn and perform any new tasks
that they are not originally designed for. This means that they can perform literally any task. So versatile is
the best fit for the first blank. For the second word, you’d want something like the “model,” in the sense of
“explain,” that you saw earlier in the sentence. Explicate is the best match.

37. Heuristic, represent. The first word is defined in the sentence as a device useful for inciting and guiding
discovery. That’s precisely what heuristic means. If you are not meant to take the model of the psyche as
guide to the physical structure of the brain, then you want a word that means something like “show” or
“depict” for the second word. Represent is the closest.

38. Technical, diffident. Since the medium… lacked a voice, they limited the technology and actors didn’t speak,
so the first word should be something like “technological.” Technical will do. For the second word, you’d
like an adjective that would explain why a character would fail to speak, perhaps something like “mute.”
Diffident means something like “self-effacing, very shy.” Reluctant is a bit of a trap answer — it does not
mean “reluctant to speak” (the similar-sounding reticent does, and the two words are often confused).

39. Mordant, disquieteing. The first word should match “bitter and ironic,” and since he got too personal and
bitter about his wife, it is evident that audience would have not enjoyed listening to their personal matters
and so disquieted which means uncomfortable suits well.

40. Attest to, quotidian. Just as… so too… suggests that the mainstream novels show a gift, just as the science
fiction novels do. The first word should mean something like “show,” or “display.” The second word should
describe the world of mainstream, rather than science-fiction, novels. You want a word that means “not
fantastic,” “ordinary,” “everyday.” Quotidian has just that meaning.

Page 48 | Text Completion | www.collegepond.com


41. Pertinent, deeply humbling. Balding is very common and it has been here from generations, yet the
scientists haven’t been able to find a cure as of now. So the topic is pertinent. Other options do not suit well.
Fully impenetrable is a trap for 2nd blank. The clue “so ignorant” is a good indication that what you want is
deeply humbling.

42. An exhaustive, remarkable variance. The blank structure for this problem is tricky; in such cases you need
to pay attention to the specific words in the sentence that give hints to the meanings of the blanks. For the
first blank, you are looking for a modifier to “comprehension.” This “comprehension,” you learn, is
impeded… by labyrinthine complexities. In other words, total, or exhaustive, comprehension is difficult
because of these complexities. For the second blank, the key word is “reliable.” The relation of this word to
the blank is given by “even the most,” telling you that these vintages are subject to something opposite to
“reliable” — that is, remarkable variance.

43. Casual nonchalance, farce. If the employee treats his “office” (a fancy way to say his “job”) with a general
lack of respect, then he treats his clients with a similarly cool and distant manner, a manner of casual
nonchalance. Note that the other two possibilities do not fit at all (meticulousness would be positive, and
envy is not indicated by the sentence). As for the second blank, knowing that “grand farce” is a common
English idiom is helpful but is not essential—the word farce, or “mockery,” is the only word that fits.

44. Initially, given teeth. For the first blank, you need something to contrast with “subsequently.” Thus, initially
fits. In the second blank, you need something that will contrast with having “little impact”. Only given teeth
has this (obviously metaphorical) meaning.

45. Veracity, consistent with. No single experiment can ever prove the “truth,” or veracity, of a theory.
Rationality is close but connotes the idea of the theory being logical, rather than factual. For the second
blank, in order for the theory to be supported, or “resilient,” the results from independent labs should agree
with each other, or be consistent with each other.

Page 49 | Text Completion | www.collegepond.com


Courses where our
Students are Admitted

- Management Information Systems - Immunology

- Computer Science - Pharmacy

- Electrical Engineering - Supply Chain Management

- Telecommunication - Aeronautical Engineering

- Biotechnology - Food Science & Technology

- Financial Engineering - Game Designing

- Architecture - Entertainment Arts & Engineering

- MEM & Tech Management - Digital Media

- Energy - Human Resources

- Civil Engineering - Economics

- Construction Management - Healthcare Systems Engineering

- Chemical Engineering - Health Administration

- Material Science - Metallurgical Engineering

- Industrial Engineering & Operations Research - Robotics

- Mechanical Engineering - Statistics

- Environmental Engineering - Textile Engineering

- Law - Psychology

022 - 2610 3135 | 022 - 2619 5832 Collegepond


info@collegepond.com www.collegepond.com
Text Completion 3 blanks drill

1. Andrew Roberts’ much lauded Napoleonic biography is especially admired as he actively avoids (i)
__________ without evidence. The book, ii) _________ titled “Napoleon: A Life”, sticks to the known facts and
does not make any (iii) _________ generalizations.

Blank(i) Blank(ii) Blank(iii)

A equivocating D aptly G sweeping

B speculating E successfully H assuaging

C prevaricating F dissonantly I disparaging

2. Coal companies often cite studies whose credibility is (i) _________ to prove that even though climate change
is real, it is not caused by the fossil fuel industry. These studies show that while the carbon dioxide content
in the atmosphere has increased, global temperatures have actually (ii) _________ over the last decade. Third
party research has concluded that these statistics are (iii) _________ at best.

Blank(i) Blank(ii) Blank(iii)

A retrospective D escalated G dubious

B garrulous E strayed H accurate

C suspect F declined I neutral

3. The fact that ransomware leaves a trace to the server, used for nefarious purposes, is being (i) _________ by
security companies. Though these traces are (ii) _________ to your run-of-the-mill antivirus, they can be (iii)
_________ and investigated by specialized anti-ransomware programs.

Blank(i) Blank(ii) Blank(iii)

A compromised D invisible G detected

B handled E moribund H deleted

C exploited F invincible I beguiled

4. Mark Twain’s writing was groundbreaking as he (i) _________ language, (ii) _________ customs, and (iii)
_________ a tapestry of themes.

Blank(i) Blank(ii) Blank(iii)

A replicated D canonized G minced

B borrowed E redefined H wove

C reinvented F refined I piqued

Page 50 | Text Completion | www.collegepond.com


5. Even though domesticating the dog was one of the most important factors leading to the development of
animal husbandry, it has largely been (i) _________ in teaching. This is not due to a lack of (ii) _________ in
research; the importance of the dog has been systematically (iii) _________ from books about the subject.

Blank(i) Blank(ii) Blank(iii)

A ignored D jurisprudence G expunged

B overestimated E diligence H included

C advanced F paradigm I ossified

6. People who expect (i) _________ are advised to stay clear of his films; his ideology is virtually (ii) _________, and
critics pounce upon this lack of (iii) _________.

Blank(i) Blank(ii) Blank(iii)

A extravagance D dormant G feebleness

B luridness E auxiliary H nuance

C subtlety F omnipresent I courage

7. The interviewer was (i) __________ by the writer’s silence. He had been led to believe that the writer would
talk about a (ii) __________ of subjects; but she simply sat there, wearing a (iii)__________ expression.

Blank(i) Blank(ii) Blank(iii)

A petrified D conflagration G nonchalant

B nonplussed E plethora H eager

C infuriated F turbulence I enthused

8. The news that Venezuela is privatizing water and electricity supply has come as a complete (i) ________. Just
last week, the prime minister was ii) _________ insistent that no such measure would be taken. Economists
have been quick to term this development deeply (iii) _________.

Blank(i) Blank(ii) Blank(iii)

A centurion D felicitously G unnerving

B coagulation E truculently H supercilious

C stupefaction F obdurately I sententious

9. Instead of (i)_________ the demand for surgeons (like other automated industries that often (ii) _________
workers) the (iii) _________ of robots which perform surgery has increased the necessity of surgeons capable
of operating them.

Page 51 | Text Completion | www.collegepond.com


Blank(i) Blank(ii) Blank(iii)

A decreasing D abandon G congé

B elevating E marginalise H novelty

C stemming F intrigue I advent

10. There is a bit of ethical (i) _________ involved in defending the illegal downloading of video games. The
principal defense is the indisputable fact that video game publishers actively (ii) _________ the developers by
mandating an unjustifiably large publisher profit fraction. However, robbing both the developer and the
publisher- which is the only outcome of piracy- is by the same token (iii) _________.

Blank(i) Blank(ii) Blank(iii)

A volatility D contort G unscrupulous

B volatility E posit H conscientious

C muckraking F fleece I felicitous

11. The poet’s first anthology exhibited a (i) _________of themes; the second anthology, on the other hand,
demonstrates a curiously (ii) _________ collection of poems unified by a (iii) _________ theme.

Blank(i) Blank(ii) Blank(iii)

A distrait D jocose G fetid

B peccadillo E consistent H lachrymose

C medley F phlegmatic I solitary

12. The wooden bridge was intended as (i) _________ till the mayor approved the new cantilever bridge further
down the river. ii) _________, the wooden bridge is now seen as (iii) _________, and the cantilever bridge is
nowhere to be found.

Blank(i) Blank(ii) Blank(iii)

A a paean D eventually G iconic

B an expedient E logically H offal

C an iota F ironically I pedestrian

13. Crusty and egotistical, the movie producer wore ___________ suits from the most exclusive tailors; was a
connoisseur of music, food, and wine; nurtured a garden of exotic plants; and enjoyed rare cigars. His self-
assured,___________ manner, together with his insistence on the best of everything, shaped his ostentatious
production house’s image as a____________ of works of enduring value.

Page 52 | Text Completion | www.collegepond.com


Blank(i) Blank(ii) Blank(iii)

A expensive D decorous G connoisseur

B immaculate E irascible H purveyor

C flamboyant F congenial I solicitor

Page 53 | Text Completion | www.collegepond.com


Text Completion 3 blanks drill solutions

1. Answers – BDG. The Clue in the sentence is that the book sticks to known facts. Therefore, it avoids option
B speculation, and does not make any option G sweeping generalizations. And which is why the book is
option D aptly titled what it’s named.

2. Answers – CFG. By elimination we get option C. Credibility can’t be retrospective or garrulous. If the studies
are doubtful, then they’ll try to prove that the global temperatures have option F declined. Which is why 3 rd
party research will conclude that such stats are option G dubious.

3. Answers – CDG. We have transition words in the 2nd sentence that will help us get the answers. If the traces
can be investigated, then the only word to fit for blank 3 is option G detected. Because of the transition
word Though, we know that blank 2 will be the opposite of blank 3. So, option D invisible. And since
negative things are happening because of these traces, the security companies will option C exploit them.

4. Answers – CEH. Since Twain’s writing was groundbreaking, we will only look for positive words.

5. Answers – AEG. Even though it was an important factor, it has been left out from teaching. So, option A. This
isn’t because of lack of dedication – option E diligence – but because the dog’s importance has been
removed from the books. So, option G.

6. Answers – CFH. The 1st and 3rd blanks need to be synonyms. So, options C & H. His ideology is obvious in his
movies. So, option F.

7. Answers – BEG. The interviewer was made to believe that the writer would talk about a variety option E of
subjects. But she didn’t. So, the interviewer has be shocked – option B. The writer just sat there with a
careless expression option G.

8. Answers CFG. The news has come as a shock because last week the minister was stubbornly saying no such
measure will take place. This has unsettled the economists.

9. Answers – AEI. The ideas in blank 1 and 3 are opposites. So, blank 1 has to mean decreasing the demand. If
such industries are decreasing the demands, then they are marginalizing workers. With the
increase/advent of robots necessity of surgeons has also increased.

10. Answers – BFG. The overall meaning of the sentence says that it is wrong to defend illegal downloading.
Therefore, option B. The publishers are over-charging the developers. So, option F. But, the sentence also
says it is wrong to rob both the parties involved. Therefore, option G.

11. Answers – CEI. The 1st and 2nd blanks need to be opposites. The only option set that is contrasting is C and
E. And then option I automatically follows.

12. Answers – BFG. The wooden bridge was only meant as a substitute or a make-do bridge till the cantilever
bridge was made. So, option B. But, what happened is the cantilever bridge is nowhere to be found and the
wooden bridge has become famous. And this is a paradox. So, options F and G.

13. Choices C, E, and H are correct. Look at the second blank first. Because of the key words Crusty and
egotistical, it would appear that Choice E, the word irascible, meaning “hot-tempered,” would fit the second
blank. And since there were so many things he had as described in the first sentence, Choice H, purveyor
(one who spreads things throughout) would be a good fit. Because of the word ostentatious (showy) in the
second sentence, you can see that Choice C, flamboyant (showy), would fit the first blank.

Page 54 | Text Completion | www.collegepond.com


What
our
students Tanaya Seth
Carnegie Mellon University
Shreesha Kulkarni
Georgia Institute of Technology

say The Collegepond team took care of everything


right from career counselling till pre-departure
tips. The SOP, LOR & Resume teams extended
Right from preparing for the GRE to the onset of
the Master’s jourey, Collegepond has played a
pivotal role in this entire process. The Converge

about phenomenal support to me. They proofread my


SOP and everything worked out very nicely. Not
only did they help me get everything ready within
platform, extensive study material, and abundant
resources, coupled with the extremely friendly,
supportive, dedicated, easily accessible and

us my deadlines, but also got me into my dream


university, CMU! It's truly amazing how much
effort the CP team puts in to suit your aspirations.
well-informed fleet of expert counselors will surely
ensure admission into one's dream university. I
would like to thank everyone at Collegepond!

Parthvi Shah John Britto Nadar Qutub Khan


New York University University of Massacchussetts, Amherst University of California, Berkeley

The different services that Collegepond offers, I had a lovely experience with Jimeet Sir and the I approached Collegepond at a rush hour, a total
have helped me a lor. They just managed to help rest of the Collegepond team. You just have to race against the clock! Jimeet and the entire team
me get rid of all my worries and apprehensions trust them; they will make your application a cut have been very supportive. They guided me
regarding my application. The way SOPs and above perfection. The entire team, from LOR pretty well during the entire process and helped
LORs were crafted, was brilliant. Counselors are counselors to resume counselors, to SOP widen my overall perspective about my master’s
always on their feet to help us with whatever counselors and of course Jimeet Sir, all have been degree. The counselors were great, always willing
queries we had. Collegepond has a well-de- instrumental and very helpful in this entire to help, and their assistance and guidance was
signed process at every step and it was a great procedure. Kudos to the entire Collegepond the key to helping me bag an admit into my dream
experience to be a beneficiary of their services. team! university. Thank you Collegepond!

Talha Kapadia Mehul Zawar


Dartmouth College Purdue University

Collegepond has helped me with everything from Being a working professional, I used to get less
Letters of Recommendation, Statement of time for researching universities, working on
Purpose to my CV and the VISA process. Their LORs and SOPs. Collegepond guided me in
online portal Converge is a one-stop solution for a each phase of this process from SOPs, LORs to
student’s counselling needs. It helped me stay in VISA mock interviews and pre-departure process-
touch with my counselors very easily and saved es. I can vouch for Collegepond, that students will
me a lot of time. Their services will only get better get the best from them in terms of service
and I am happy to call myself a CP student. throughout the application process!

022 - 2610 3135 | 022 - 2619 5832 Collegepond


info@collegepond.com www.collegepond.com

You might also like